SlideShare uma empresa Scribd logo
1 de 91
Baixar para ler offline
Dr. Ole Peter Smith
                                      Instituto de Matemática e Estatística
                                         Universidade Federal de Goiás
                              ole@mat.ufg.br - http://www.ime.ufg.br/docentes/olepeter


                      Data:                    15/04/2009
                      Semestre:
                      Curso:                   Engenharia de Computação
                      Disciplina:              Álgebra Linear
                      Prova:                   I
1. 4 pts. Dado as matrizes:             0               1                   0                1
                                        1       −2    0                    −1         0    0
                                    A=@ 0        1    1 A              B=@ 0          2    0 A
                                        1       −2    1                    0          0    3
     (a) Encontrar: det A e det B.
     (b) Encontrar as matrizes adjuntas: A∗ e B ∗ .
     (c) Encontrar os inversos: A−1 e B −1 .
     (d) Encontrar os determinantes dos inversos: det (A−1 ) e det (B −1 ).
     (e) Encontrar os produtos: A B e B A.
     (f) Mostre que em geral vale por matrizes do mesmo ordem: (A B)−1 = B −1 A−1 .
     (g) Encontrar o inverso do produto: (A B)−1 .
     (h) Encontrar o inverso do produto: (B A)−1 .
   Solution:

     (a)                                     ˛                   ˛ ˛             ˛
                                             ˛ 1
                                             ˛     −2 0          ˛ ˛ 1 −2 0 ˛
                                                                 ˛ ˛             ˛
                                     det A = ˛ 0
                                             ˛       1   1       ˛=˛ 0
                                                                 ˛ ˛      1   1 ˛=1·1·1=1
                                                                                 ˛
                                             ˛ 1   −2 1          ˛ ˛ 0    0   1 ˛
                                                    ˛                ˛
                                                    ˛ −1         0 0 ˛
                                                    ˛                ˛
                                            det B = ˛ 0
                                                    ˛            2 0 ˛ = (−1) · 2 · 3 = −6
                                                                     ˛
                                                    ˛ 0          0 3 ˛
     (b) Para A:
                              ˛        ˛                    ˛      ˛                            ˛        ˛
                              ˛ 1    1 ˛                    ˛ 0 1 ˛                             ˛ 0   1 ˛
                        D11 = ˛
                              ˛ −2 1 ˛ = 3                  ˛ 1 1 ˛ = −1
                                                      D12 = ˛                                   ˛ 1 −2 ˛ = −1
                                                                                          D13 = ˛
                                       ˛                           ˛                                     ˛
                                ˛        ˛                     ˛     ˛                            ˛        ˛
                                ˛ −2 0 ˛                       ˛ 1 0 ˛                            ˛ 1 −2 ˛
                        D21 = ˛ ˛ −2 1 ˛ = −2           D22 = ˛˛ 1 1 ˛=1                  D23 = ˛ ˛ 1 −2 ˛ = 0
                                         ˛                           ˛                                     ˛
                                ˛        ˛                     ˛     ˛                            ˛        ˛
                                ˛ −2 0 ˛                       ˛ 1 0 ˛                            ˛ 1 −2 ˛
                        D31 = ˛ ˛        ˛ = −2         D32 = ˛˛     ˛=1                  D33 = ˛ ˛        ˛=1
                                  1   1 ˛                        0 1 ˛                              0  1 ˛
           Assim:
                                                A11 = 3      A12 = 1       A13 = −1
                                                   A21 = 2       A22 = 1   A23 = 0
                                                A31 = −2 A32 = −1 A33 = 1
           Ou seja:                                                0               1
                                                                 3          2   −2
                                                A∗ = (Aji ) = @ 1           1   −1 A
                                                                −1          0    1
           Por B ser diagonal:
                                                       Dij = Aij = 0, i = j




                                                             1                   Quem entende sério somente por sério
                                                                                 E brincadeira somente por brincadeira
                                                                                De fato desentendeu ambos... Piet Hein
Dr. Ole Peter Smith
                                             Instituto de Matemática e Estatística
                                                Universidade Federal de Goiás
                               ole@mat.ufg.br - http://www.ime.ufg.br/docentes/olepeter



                         ˛           ˛                                         ˛              ˛                                    ˛           ˛
                         ˛ 2       0 ˛                                         ˛ −1         0 ˛                                    ˛ −1      0 ˛
           D11 = A11    =˛           ˛=6                   D22 = A22          =˛              ˛ = −3                 D33 = A33    =˛           ˛ = −2
                         ˛ 0       3 ˛                                         ˛ 0          3 ˛                                    ˛ 0       2 ˛

           Ou seja:                                                           0                 1
                                                                         6              0    0
                                                                      ∗
                                                                    B =@ 0             −3     0 A
                                                                         0              0    −2
     (c) Encontrar os inversos: A−1 e B −1 .
                                                                              0                               1
                                                                                 3                2        −2
                                                          −1           1    ∗
                                                       A           =       A =@ 1                 1        −1 A
                                                                     det A
                                                                                −1                0         1
           E:                                                                 0                               1
                                                                                −1                    0     0
                                                              −1       1    ∗                         1
                                                          B        =       B =@ 0                     2
                                                                                                            0 A
                                                                     det B                                  1
                                                                                0                     0     3

     (d)
                                                                                          1
                                                                     det (A−1 ) =             =1
                                                                                        det A
                                                                                         1      1
                                                                    det (B −1 ) =            =−
                                                                                       det B    6
     (e)                                         0                    10                       1 0                            1
                                         1 −2                       0             −1    0    0                   1    −4    0
                                   AB =@ 0  1                       1 A@          0     2    0 A=@               0    2     3 A
                                         1 −2                       1             0     0    3                  −1    −4    3
                                       0                              10                       1 0                            1
                                         −1 0                       0             1    −2    0                  −1     2    0
                                   BA=@ 0    2                      0 A@          0     1    1 A=@               0    2     2 A
                                          0  0                      3             1    −2    1                   3    −6    3
     (f) Por o inverso ser único e o produto de matrizes ser associativo:

                                    (B −1 A−1 )(A B) = B −1 (A−1 A)B = B −1 I B = B −1 B = I

           E:
                                    (A B) (B −1 A−1 ) = A(B B −1 )A−1 = A I A−1 = A A−1 = I

                                                                                                                                                 QED

     (g)                                                  0                     10                           1 0                          1
                                                        −1             0      0     3             2       −2       −3            −2    2
                              −1        −1    −1                       1
                      (A B)        =B        A       =@ 0              2
                                                                              0 A@ 1              1       −1 A = @ 12
                                                                                                                                 1
                                                                                                                                 2
                                                                                                                                      −21 A
                                                                              1
                                                        0              0      3
                                                                                   −1             0        1       −13
                                                                                                                                 0     1
                                                                                                                                       3

     (h) Encontrar o inverso do produto: (B A)−1 .

                                                                                                                                      −2
                                                          0                      10                           1 0                        1
                                                          3               2   −2     −1               0     0       −3           1     3
                              −1        −1       −1                                                   1                          1     1 A
                      (B A)        =A        B        =@ 1                1   −1 A @ 0                2
                                                                                                            0 A = @ −1           2
                                                                                                                                      −3
                                                                                                            1                         1
                                                         −1               0    1      0               0     3
                                                                                                                     1           0    3

2. 4 pts. Dado a sistema linear:
                                    8                                                                                        9
                                    < x1              +       2x2     +       3x3      +    4x4       +      5x5     =     1 =
                              (∗) :   2x1             +       3x2     +       4x3      +    5x4       −       x5     =     2
                                      3x1             +       4x2     +       5x3      +    6x4       −      3x5     =     3
                                    :                                                                                        ;



                                                                              2                            Quem entende sério somente por sério
                                                                                                           E brincadeira somente por brincadeira
                                                                                                          De fato desentendeu ambos... Piet Hein
Dr. Ole Peter Smith
                                       Instituto de Matemática e Estatística
                                          Universidade Federal de Goiás
                             ole@mat.ufg.br - http://www.ime.ufg.br/docentes/olepeter



     (a) Encontrar a solução completa do sistema homogêneo do (∗).
    (b) Encontrar a solução completa do sistema não homogêneo.
     (c) Encontrar a solução completa do sistema:
                                    8                                                                                 9
                                    < x1 + 2x2                     +       3x3        +       4x4     +   5x5   =   2 =
                             (∗∗) :    2x1 + 3x2                   +       4x3        +       5x4     −    x5   =   3
                                       3x1 + 4x2                   +       5x3        +       6x4     −   3x5   =   4
                                    :                                                                                 ;

   Solution:
   Resolvemos tudo de uma vez:
                                            0                                                     1
                                              1       2    3       4        5     |       1     2
                                            @ 2       3    4       5       −1     |       2     3 A∼
                                              3       4    5       6       −3     |       3     4
                                       0                                                              1
                                         1       2        3         4        5        |       1   2
                                       @ 0      −1        −2       −3       −11       |       0 −1 A ∼
                                         0      −2        −4       −6       −18       |       0 −2
                                        0                                                           1
                                           1 0 −1 −2 −17 |                                    1 0
                                         @ 0 1   2   3   11  |                                0 1 A∼
                                           0 0   0   0   −4 |                                 0 0
                                           0                                                        1
                                             1 0 −1 −2 0 |                                    1 0
                                           @ 0 1   2   3   0 |                                0 1 A
                                             0 0   0   0   1 |                                0 0
   Pondo x3 = t e x4 = s obtemos a solução completa do sistema homogêneo:

                              (x1 , x2 , x3 , x4 , x5 )T = (t + 2s, −2t − 3s, t, s, 0)T , (t, s) ∈ R2
   E por (∗):

                            (x1 , x2 , x3 , x4 , x5 )T = (1 + t + 2s, −2t − 3s, t, s, 0)T , (t, s) ∈ R2
   E por (∗∗):

                             (x1 , x2 , x3 , x4 , x5 )T = (t + 2s, 1 − 2t − 3s, t, s, 0)T , (t, s) ∈ R2
   Comentário: Verificar:
   (i) Verifique que a solução da eq. homogênea satisfaz as eqs.
   (ii) Verifique que as soluções particulares satisfaz (∗) resp. (∗∗).
3. 2 pts. (Cabeludo) Dado as matrizes:
                                                  0                           1
                                               1               2       3    4
                                             B 4               3       2    1 C
                                           A=B                                C , (a, b) ∈ R2
                                             @ a               2       3    4 A
                                               4               3       2    b
                                               0                                1
                                                  0                a       0 0
                                                B a                0       a 0 C
                                              B=@
                                                B                               C, a ∈ R
                                                  0                a       0 a A
                                                  0                0       a 0
     (a) Encontrar para quaisquer valores de a e b o determinante do A.
    (b) Encontrar para quaisquer valores de a e b o posto do A.
     (c) Encontrar para qualquer valor de a o determinante do B.


                                                                   3                                 Quem entende sério somente por sério
                                                                                                     E brincadeira somente por brincadeira
                                                                                                    De fato desentendeu ambos... Piet Hein
Dr. Ole Peter Smith
                                               Instituto de Matemática e Estatística
                                                  Universidade Federal de Goiás
                               ole@mat.ufg.br - http://www.ime.ufg.br/docentes/olepeter



 (d) Para quais valores de a o matriz B tem inverso? Para estes valores, encontrar o inverso.
Solution:

 (a)                                               ˛                                ˛ ˛                        ˛
                                                   ˛
                                                   ˛      1       2       3       4 ˛ ˛ 1
                                                                                    ˛ ˛           2 3      4 ˛ ˛
                                                   ˛      4       3       2       1 ˛ ˛ 4         3 2      1 ˛
                                           det A = ˛                                ˛=˛
                                                                                    ˛ ˛ a−1 0 0
                                                                                                               ˛=
                                                   ˛
                                                   ˛      a       2       3       4 ˛ ˛                    0 ˛ ˛
                                                   ˛      4       3       2       b ˛ ˛ 0         0 0 b−1 ˛
                                                              ˛                     ˛
                          3+1          4+4
                                                              ˛ 2                 3 ˛
                   (−1)         (−1)           (a − 1)(b − 1) ˛
                                                              ˛ 3
                                                                                    ˛ = (a − 1)(b − 1)(4 − 9) = −5(a − 1)(b − 1)
                                                                                  2 ˛
 (b) Como no item anterior:                     0                           1 0                                                 1
                                                  1       2       3       4      1                          2       3        4
                                                B 4       3       2       1 C B 4                           3       2        1 C
                                                B                           C∼B                                                 C
                                                @ a       2       3       4 A @ a−1                         0       0        0 A
                                                  4       3       2       b      0                          0       0       b−1
       Donde segue:
                                                                       8
                                                                       > 4,
                                                                       >                    a=1∧b=1
                                                                         3,                 a=1∧b=1
                                                                       <
                                                                  ρA =
                                                                       > 3,
                                                                       >                    a=1∧b=1
                                                                         2,                  a=b=1
                                                                       :

 (c)
               ˛                           ˛   ˛                                  ˛   ˛                                     ˛   ˛                            ˛   ˛                   ˛
               ˛
               ˛   0   a       0       0   ˛
                                           ˛
                                               ˛
                                               ˛      0       1       0       0   ˛
                                                                                  ˛
                                                                                      ˛
                                                                                      ˛         0       1       0       0   ˛
                                                                                                                            ˛
                                                                                                                                ˛
                                                                                                                                ˛            1   0   0   0   ˛
                                                                                                                                                             ˛
                                                                                                                                                                 ˛
                                                                                                                                                                 ˛   1   0   0   0   ˛
                                                                                                                                                                                     ˛
               ˛   a   0       a       0   ˛  4˛
                                               ˛      1       0       1       0   ˛  4˛
                                                                                      ˛         1       0       0       0   ˛  4˛
                                                                                                                                ˛            0   1   0   0   ˛  4˛
                                                                                                                                                                 ˛   0   1   0   0   ˛ = a4
                                                                                                                                                                                     ˛
       det B = ˛                           ˛=a                                    ˛=a                                       ˛=a                              ˛=a
               ˛
               ˛   0   a       0       a   ˛
                                           ˛
                                               ˛
                                               ˛      0       1       0       1   ˛
                                                                                  ˛
                                                                                      ˛
                                                                                      ˛         0       0       0       1   ˛
                                                                                                                            ˛
                                                                                                                                ˛
                                                                                                                                ˛            0   0   0   1   ˛
                                                                                                                                                             ˛
                                                                                                                                                                 ˛
                                                                                                                                                                 ˛   0   0   1   0   ˛
                                                                                                                                                                                     ˛
               ˛   0   0       a       0   ˛   ˛      0       0       1       0   ˛   ˛         0       0       1       0   ˛   ˛            0   0   1   0   ˛   ˛   0   0   0   1   ˛

 (d) O inverso existe se e somente se o determinante não for zero:                              a4 = 0              ⇔           a = 0.
     Por a = 0 obtemos:
                0                                     1 0                                                                                               1
                    0 a 0 0 | 1 0 0 0                           0                                   a       0       0       |        1       0   0    0
                B a 0 a 0 | 0 1 0 0 C B a                                                           0       0       0       |        0       1   0   −1 C
                @ 0 a 0 a | 0 0 1 0 A∼@ 0
                B                                     C B                                                                                               C∼
                                                                                                    0       0       a       |       −1       0   1    0 A
                    0 0 a 0 | 0 0 0 1                           0                                   0       a       0       |        0       0   0    1

                                                                                                                                                 1         1
                0                                                            1 0                                                                             1
                  a        0       0       0   |     0        1       0   −1     1                      0       0       0       |        0       a
                                                                                                                                                     0   −a
                                                                                                                                         1
                B 0
                B          a       0       0   |     1        0       0    0 C B 0
                                                                             C∼B                        1       0       0       |        a
                                                                                                                                                 0   0    0 CC
                                                                                                                                                          1
                @ 0        0       a       0   |     0        0       0    1 A @ 0                      0       1       0       |     0          0   0    a
                                                                                                                                                             A
                                                                                                                                       1             1
                  0        0       0       a   |    −1        0       1    0     0                      0       0       1       |    −a          0   a
                                                                                                                                                          0
       Assim:

                                                                                            1                 1
                                                                          0                                     1
                                                                                       0    a
                                                                                                    0       −a
                                                                                       1
                                                                                            0       0        0 C
                                                              B −1 = B
                                                                     B
                                                                                       a                        C
                                                                                                             1
                                                                     @                 0    0       0        a
                                                                                                                A
                                                                                        1           1
                                                                                      −a    0       a
                                                                                                             0




                                                                                  4                              Quem entende sério somente por sério
                                                                                                                 E brincadeira somente por brincadeira
                                                                                                                De fato desentendeu ambos... Piet Hein
Dr. Ole Peter Smith
                                         Instituto de Matemática e Estatística
                                            Universidade Federal de Goiás
                               ole@mat.ufg.br - http://www.ime.ufg.br/docentes/olepeter



                     Data:                        05/05/2009
                     Semestre:
                     Curso:                       Engenharia Civil
                     Disciplina:                  Álgebra Linear
                     Prova:                       I
1. 4 pts. Dado as matrizes:                0                1               0             1
                                         1         −2     0                  −1     0   0
                                     A=@ 0          1     1 A            B=@ 0      2   0 A
                                         1         −2     1                  0      0   3
   Sabendo que:

                                                                              1
                                                  det (A−1 ) = det A−1 =
                                                                            det A
   e:

                                                         (A B)−1 = B −1 A−1
   responde o seguinte:

        (a) Encontrar: det A e det B.
        (b) Encontrar as matrizes adjuntas: A∗ e B ∗ .
        (c) Encontrar os inversos: A−1 e B −1 .
        (d) Encontrar os determinantes dos inversos: det (A−1 ) e det (B −1 ).
        (e) Encontrar os produtos: A B e B A.
        (f) Encontrar o inverso do produto: (A B)−1 .

   Solution:
   4 pts. Dado as matrizes:                0                1               0             1
                                         1         −2     0                  −1     0   0
                                     A=@ 0          1     1 A            B=@ 0      2   0 A
                                         1         −2     1                  0      0   3
   Sabendo que:

                                                                              1
                                                  det (A−1 ) = det A−1 =
                                                                            det A
   e:

                                                         (A B)−1 = B −1 A−1
   responde o seguinte:

        (a) Encontrar: det A e det B.
                                                ˛                   ˛ ˛             ˛
                                                ˛ 1
                                                ˛     −2 0          ˛ ˛ 1 −2 0 ˛
                                                                    ˛ ˛             ˛
                                        det A = ˛ 0
                                                ˛       1   1       ˛=˛ 0
                                                                    ˛ ˛      1   1 ˛=1·1·1=1
                                                                                    ˛
                                                ˛ 1   −2 1          ˛ ˛ 0    0   1 ˛
                                                       ˛                ˛
                                                       ˛ −1         0 0 ˛
                                                       ˛                ˛
                                               det B = ˛ 0
                                                       ˛            2 0 ˛ = (−1) · 2 · 3 = −6
                                                                        ˛
                                                       ˛ 0          0 3 ˛




                                                                5                         I really do like Your Christ
                                                                                     But I do dislike Your Christians
                                             Why can’t Your Christians be more like Your Christ? - Mahatma Gandhi
Dr. Ole Peter Smith
                                 Instituto de Matemática e Estatística
                                    Universidade Federal de Goiás
                       ole@mat.ufg.br - http://www.ime.ufg.br/docentes/olepeter



(b) Encontrar as matrizes adjuntas: A∗ e B ∗ .
    Para A:
                      ˛        ˛                        ˛                ˛                        ˛         ˛
                      ˛ 1    1 ˛                        ˛ 0            1 ˛                        ˛ 0    1 ˛
                D11 = ˛
                      ˛        ˛=3                D12 = ˛                ˛ = −1             D13 = ˛
                                                                                                  ˛         ˛ = −1
                          −2 1 ˛                        ˛ 1            1 ˛                            1 −2 ˛
                        ˛        ˛                         ˛                ˛                       ˛         ˛
                        ˛ −2 0 ˛                           ˛          1 0 ˛                         ˛ 1 −2 ˛
                D21 = ˛ ˛ −2 1 ˛ = −2               D22 = ˛                 ˛=1             D23 = ˛ ˛ 1 −2 ˛ = 0
                                 ˛                                                                            ˛
                                                           ˛          1 1 ˛
                        ˛        ˛                         ˛                ˛                       ˛         ˛
                        ˛ −2 0 ˛                           ˛          1 0 ˛                         ˛ 1 −2 ˛
                D31 = ˛ ˛        ˛ = −2             D32 = ˛                 ˛=1             D33 = ˛ ˛         ˛=1
                           1  1 ˛                          ˛          0 1 ˛                            0  1 ˛
    Assim:
                                              A11 = 3    A12 = 1           A13 = −1
                                               A21 = 2       A22 = 1       A23 = 0
                                              A31 = −2 A32 = −1 A33 = 1
    Ou seja:                                                      0                     1
                                                               3             2       −2
                                              A∗ = (Aji ) = @ 1              1       −1 A
                                                              −1             0        1
    Por B ser diagonal:
                                                   Dij = Aij = 0, i = j

                 ˛          ˛                             ˛                  ˛                              ˛          ˛
                 ˛ 2      0 ˛                             ˛ −1             0 ˛                              ˛ −1     0 ˛
     D11 = A11 = ˛          ˛=6               D22 = A22 = ˛                  ˛ = −3             D33 = A33 = ˛          ˛ = −2
                 ˛ 0      3 ˛                             ˛ 0              3 ˛                              ˛ 0      2 ˛
    Ou seja:                                             0                     1
                                                       6               0    0
                                                   ∗
                                                  B =@ 0              −3     0 A
                                                       0               0    −2
(c) Encontrar os inversos: A−1 e B −1 .
                                                            0                            1
                                                               3                 2    −2
                                           −1        1    ∗
                                          A      =       A =@ 1                  1    −1 A
                                                   det A
                                                              −1                 0     1
    E:                                                        0                          1
                                                                −1               0     0
                                                     1
                                          B −1   =       B∗ = @ 0                1
                                                                                 2
                                                                                       0 A
                                                   det B                               1
                                                                0                0     3

(d) Encontrar os determinantes dos inversos: det (A−1 ) e det (B −1 ).

                                                                         1
                                                  det (A−1 ) =               =1
                                                                       det A
                                                                        1      1
                                                 det (B −1 ) =              =−
                                                                      det B    6
(e) Encontrar os produtos: A B e B A.
                                  0                 10                        1 0                       1
                                1 −2              0           −1       0    0               1    −4   0
                          AB =@ 0  1              1 A@        0        2    0 A=@           0    2    3 A
                                1 −2              1           0        0    3              −1    −4   3
                              0                     10                        1 0                       1
                                −1 0              0           1       −2    0              −1     2   0
                          BA=@ 0    2             0 A@        0        1    1 A=@           0    2    2 A
                                 0  0             3           1       −2    1               3    −6   3


                                                         6                         I really do like Your Christ
                                                                              But I do dislike Your Christians
                                      Why can’t Your Christians be more like Your Christ? - Mahatma Gandhi
Dr. Ole Peter Smith
                                               Instituto de Matemática e Estatística
                                                  Universidade Federal de Goiás
                                 ole@mat.ufg.br - http://www.ime.ufg.br/docentes/olepeter



     (f) Encontrar o inverso do produto: (A B)−1 .
                                                         0                10                   1 0                             1
                                                        −1          0   0     3         2   −2       −3           −2        2
                             −1           −1    −1                  1
                    (A B)         =B           A     =@ 0           2
                                                                        0 A@ 1          1   −1 A = @ 12
                                                                                                                  1
                                                                                                                  2
                                                                                                                           −21 A
                                                                        1
                                                        0           0   3
                                                                             −1         0    1       −13
                                                                                                                  0        1
                                                                                                                           3

2. 4 pts. Dado a sistema linear:
                                   8                                                                          9
                                   < x1              +       2x2   +    3x3   +   4x4       +   5x5   =     1 =
                             (∗) :   2x1             +       3x2   +    4x3   +   5x4       −    x5   =     2
                                     3x1             +       4x2   +    5x3   +   6x4       −   3x5   =     3
                                   :                                                                          ;

     (a) Encontrar a solução completa do sistema homogêneo do (∗).
     (b) Encontrar a solução completa do sistema não homogêneo.
     (c) Encontrar a solução completa do sistema:
                                    8                                                                            9
                                    < x1 + 2x2                          +   3x3   +   4x4       +   5x5    =   2 =
                             (∗∗) :    2x1 + 3x2                        +   4x3   +   5x4       −    x5    =   3
                                       3x1 + 4x2                        +   5x3   +   6x4       −   3x5    =   4
                                    :                                                                            ;

   Solution:
   4 pts. Dado a sistema linear:
                                   8                                                                          9
                                   < x1              +       2x2   +    3x3   +   4x4       +   5x5   =     1 =
                             (∗) :   2x1             +       3x2   +    4x3   +   5x4       −    x5   =     2
                                     3x1             +       4x2   +    5x3   +   6x4       −   3x5   =     3
                                   :                                                                          ;

     (a) Encontrar a solução completa do sistema homogêneo do (∗).
     (b) Encontrar a solução completa do sistema não homogêneo.
     (c) Encontrar a solução completa do sistema:
                                    8                                                                            9
                                    < x1 + 2x2                          +   3x3   +   4x4       +   5x5    =   2 =
                             (∗∗) :    2x1 + 3x2                        +   4x3   +   5x4       −    x5    =   3
                                       3x1 + 4x2                        +   5x3   +   6x4       −   3x5    =   4
                                    :                                                                            ;

   Resolvemos tudo de uma vez:
                   0                                 ˛             1    0                                 ˛           1
                     1       2        3    4   5     ˛ 1
                                                     ˛    2              1  2   3  4  5                   ˛ 1
                                                                                                          ˛     2
                   @ 2       3        4    5   −1    ˛ 2
                                                     ˛    3        A ∼ @ 0 −1 −2 −3 −11                   ˛ 0 −1
                                                                                                          ˛
                                                                                                                      A∼
                     3       4        5    6   −3    ˛ 3  4              0 −2 −4 −6 −18                   ˛ 0 −2
                    0                                     ˛             1 0                                 ˛          1
                         1        0       −1   −2     −17 ˛
                                                          ˛        1 0        1 0 −1 −2                   0 ˛ 1 0
                                                                                                            ˛
                       @ 0        1        2   3       11 ˛
                                                          ˛        0 1 A∼@ 0 1     2 3                    0 ˛ 0 1
                                                                                                            ˛
                                                                                                                       A
                         0        0        0   0      −4 ˛         0 0        0 0  0 0                    1 ˛ 0 0




                                                                        7                       I really do like Your Christ
                                                                                           But I do dislike Your Christians
                                                   Why can’t Your Christians be more like Your Christ? - Mahatma Gandhi
Dr. Ole Peter Smith
                                            Instituto de Matemática e Estatística
                                               Universidade Federal de Goiás
                               ole@mat.ufg.br - http://www.ime.ufg.br/docentes/olepeter



   Pondo x3 = t e x4 = s obtemos a solução completa do sistema homogêneo:
                           0        1        0         1        0               1     0                  1
                               x1                  1                 2                      t + 2s
                           B   x2   C    B        −2   C    B       −3          C B        −2t − 3s      C
                                                                                                         C , (t, s) ∈ R2
                           B        C    B             C    B                   C B                      C
                           B
                           B   x3   C = tB
                                    C    B         1   C + sB
                                                       C    B        0          C=B
                                                                                C B            t         C
                           @   x4   A    @         0   A    @        1          A @            s         A
                               x5                  0                 0                         0
   E por (∗):
                  0        1        0       1      0           1            0        1       0                1
                      x1                1               1                        2               1 + t + 2s
                  B   x2   C B          0   C    B     −2      C    B           −3   C B         −2t − 3s     C
                                                                                                              C , (t, s) ∈ R2
                  B        C B              C    B             C    B                C B                      C
                  B
                  B   x3   C=B
                           C B          0   C + tB
                                            C    B      1      C + sB
                                                               C    B            0   C=B
                                                                                     C B              t       C
                  @   x4   A @          0   A    @      0      A    @            1   A @              s       A
                      x5                0               0                        0                   0
   E por (∗∗):
                  0        1   0   1    0    1   0    1 0                                                      1
                    x1           0         1        2        t + 2s
                  B x2     C B 1 C      B −2 C   B −3 C B 1 − 2t − 3s                                          C
                                                                                                               C , (t, s) ∈ R2
                  B        C B     C    B    C   B    C B                                                      C
                  B x3     C = B 0 C + tB 1 C + sB 0 C = B      t
                  B        C B     C    B    C   B    C B                                                      C
                  @ x4     A @ 0 A      @ 0 A    @ 1 A @        s                                              A
                    x5           0         0        0           0
   Comentário: Controle:
   (i) Verifique que a solução da eqação homogênea satisfaz as eqções.
   (ii) Verifique que as soluções particulares satisfaz (∗) resp. (∗∗).
3. 2 pts. (Cabeludo) Dado as matrizes:
                                                       0                          1
                                                    1          2        3       4
                                                  B 4          3        2       1 C
                                                A=B
                                                  @ a
                                                                                  C,       (a, b) ∈ R2
                                                               2        3       4 A
                                                    4          3        2       b
                                                           0                          1
                                                      0             a       0       0
                                                    B a             0       a       0 C
                                                  B=@
                                                    B                                 C,     a∈R
                                                      0             a       0       a A
                                                      0             0       a       0
     (a) Encontrar para quaisquer valores de a e b o determinante do A.
    (b) Encontrar para quaisquer valores de a e b o posto do A.
     (c) Encontrar para qualquer valor de a o determinante do B.
    (d) Para quais valores de a o matriz B tem inverso? Para estes valores, encontrar o inverso.
   Solution:
   2 pts. (Cabeludo) Dado as matrizes:
                                                       0                          1
                                                    1          2        3       4
                                                  B 4          3        2       1 C
                                                A=B
                                                  @ a
                                                                                  C,       (a, b) ∈ R2
                                                               2        3       4 A
                                                    4          3        2       b
                                                           0                          1
                                                      0             a       0       0
                                                    B a             0       a       0 C
                                                  B=@
                                                    B                                 C,     a∈R
                                                      0             a       0       a A
                                                      0             0       a       0



                                                                    8                        I really do like Your Christ
                                                                                        But I do dislike Your Christians
                                                Why can’t Your Christians be more like Your Christ? - Mahatma Gandhi
Dr. Ole Peter Smith
                                            Instituto de Matemática e Estatística
                                               Universidade Federal de Goiás
                            ole@mat.ufg.br - http://www.ime.ufg.br/docentes/olepeter



(a) Encontrar para quaisquer valores de a e b o determinante do A.
                                                ˛                                ˛ ˛                        ˛
                                                ˛
                                                ˛      1       2       3       4 ˛ ˛ 1
                                                                                 ˛ ˛           2 3      4 ˛ ˛
                                                ˛      4       3       2       1 ˛ ˛ 4         3 2      1 ˛
                                        det A = ˛                                ˛=˛
                                                                                 ˛ ˛ a−1 0 0
                                                                                                            ˛=
                                                ˛
                                                ˛      a       2       3       4 ˛ ˛                    0 ˛ ˛
                                                ˛      4       3       2       b ˛ ˛ 0         0 0 b−1 ˛
                                                           ˛                     ˛
                       3+1          4+4
                                                           ˛ 2                 3 ˛
                (−1)         (−1)           (a − 1)(b − 1) ˛
                                                           ˛ 3
                                                                                 ˛ = (a − 1)(b − 1)(4 − 9) = −5(a − 1)(b − 1)
                                                                               2 ˛
(b) Encontrar para quaisquer valores de a e b o posto do A.
    Como no item anterior:       0                   1 0                                                                     1
                                     1 2 3 4                1                                            2       3        4
                                 B 4 3 2 1 C B 4                                                         3       2        1 C
                                 B                   C∼B                                                                     C
                                 @ a 2 3 4 A @ a−1                                                       0       0        0 A
                                     4 3 2 b                0                                            0       0       b−1
    Donde segue:
                                                                8
                                                                > 4,
                                                                >                        a=1∧b=1
                                                                  3,                     a=1∧b=1
                                                                <
                                                           ρA =
                                                                > 3,
                                                                >                        a=1∧b=1
                                                                  2,                      a=b=1
                                                                :

(c) Encontrar para qualquer valor de a o determinante do B.

            ˛                           ˛   ˛                                  ˛   ˛                                     ˛   ˛                            ˛   ˛                   ˛
            ˛
            ˛   0   a       0       0   ˛
                                        ˛
                                            ˛
                                            ˛      0       1       0       0   ˛
                                                                               ˛
                                                                                   ˛
                                                                                   ˛         0       1       0       0   ˛
                                                                                                                         ˛
                                                                                                                             ˛
                                                                                                                             ˛            1   0   0   0   ˛
                                                                                                                                                          ˛
                                                                                                                                                              ˛
                                                                                                                                                              ˛   1   0   0   0   ˛
                                                                                                                                                                                  ˛
            ˛   a   0       a       0   ˛  4˛
                                            ˛      1       0       1       0   ˛  4˛
                                                                                   ˛         1       0       0       0   ˛  4˛
                                                                                                                             ˛            0   1   0   0   ˛  4˛
                                                                                                                                                              ˛   0   1   0   0   ˛ = a4
                                                                                                                                                                                  ˛
    det B = ˛                           ˛=a                                    ˛=a                                       ˛=a                              ˛=a
            ˛
            ˛   0   a       0       a   ˛
                                        ˛
                                            ˛
                                            ˛      0       1       0       1   ˛
                                                                               ˛
                                                                                   ˛
                                                                                   ˛         0       0       0       1   ˛
                                                                                                                         ˛
                                                                                                                             ˛
                                                                                                                             ˛            0   0   0   1   ˛
                                                                                                                                                          ˛
                                                                                                                                                              ˛
                                                                                                                                                              ˛   0   0   1   0   ˛
                                                                                                                                                                                  ˛
            ˛   0   0       a       0   ˛   ˛      0       0       1       0   ˛   ˛         0       0       1       0   ˛   ˛            0   0   1   0   ˛   ˛   0   0   0   1   ˛

(d) Para quais valores de a o matriz B tem inverso? Para estes valores, encontrar o inverso.
    O inverso existe se e somente se o determinante não for zero:                            a4 = 0              ⇔           a = 0.
    Por a = 0 obtemos:
               0                                     1 0                                                                                             1
                   0 a 0 0 | 1 0 0 0                           0                                 a       0       0       |        1       0   0    0
               B a 0 a 0 | 0 1 0 0 C B a                                                         0       0       0       |        0       1   0   −1 C
               @ 0 a 0 a | 0 0 1 0 A∼@ 0
               B                                     C B                                                                                             C∼
                                                                                                 0       0       a       |       −1       0   1    0 A
                   0 0 a 0 | 0 0 0 1                           0                                 0       a       0       |        0       0   0    1

                                                                                                                                              1         1
              0                                                           1 0                                                                             1
                a       0       0       0   |    0     1       0       −1     1                      0       0       0       |        0       a
                                                                                                                                                  0   −a
                                                                                                                                      1
              B 0
              B         a       0       0   |    1     0       0        0 C B 0
                                                                          C∼B                        1       0       0       |        a
                                                                                                                                              0   0    0 CC
                                                                                                                                                       1
              @ 0       0       a       0   |    0     0       0        1 A @ 0                      0       1       0       |     0          0   0    a
                                                                                                                                                          A
                                                                                                                                    1             1
                0       0       0       a   |   −1     0       1        0     0                      0       0       1       |    −a          0   a
                                                                                                                                                       0
    Assim:

                                                                                         1                 1
                                                                       0                                     1
                                                                                    0    a
                                                                                                 0       −a
                                                                                    1
                                                                                         0       0        0 C
                                                       B −1 = B
                                                              B
                                                                                    a                        C
                                                                                                          1
                                                              @                     0    0       0        a
                                                                                                             A
                                                                                     1           1
                                                                                   −a    0       a
                                                                                                          0




                                                                               9             I really do like Your Christ
                                                                                        But I do dislike Your Christians
                                                Why can’t Your Christians be more like Your Christ? - Mahatma Gandhi
Dr. Ole Peter Smith
                                            Instituto de Matemática e Estatística
                                               Universidade Federal de Goiás
                               ole@mat.ufg.br - http://www.ime.ufg.br/docentes/olepeter



                    Data:                            27/05/2009
                    Semestre:
                    Curso:                           Engenharia de Computação
                    Disciplina:                      Álgebra Linear
                    Prova:                           II
1. 2 pts. Dado os vetores em relação ao base canônica, (e1 , e2 , e3 , e4 , e5 ), em R5 :

           v1 = (1, 1, 1, 1, 1)T           v2 = (1, −1, 1, −1, 1)T                  v3 = (3, −1, 3, −1, 3)T                        v4 = (0, 1, 0, 1, 0)T

     (a) Mostre que: V = ger(v1 , v2 ) = ger(v3 , v4 ). Qual a dimensão do V ?
     (b) Escreve v1 e v2 como combinações lineares de v3 e v4 .
     (c) Escreve v3 e v4 como combinações lineares de v1 e v2 .
   Solution:
   2 pts. Dado os vetores em relação ao base canônica, (e1 , e2 , e3 , e4 , e5 ), em R5 :

           v1 = (1, 1, 1, 1, 1)T           v2 = (1, −1, 1, −1, 1)T                  v3 = (3, −1, 3, −1, 3)T                        v4 = (0, 1, 0, 1, 0)T

     (a) Mostre que: V = ger(v1 , v2 ) = ger(v3 , v4 ). Qual a dimensão do V ?
     (b) Escreve v1 e v2 como combinações lineares de v3 e v4 .
     (c) Escreve v3 e v4 como combinações lineares de v1 e v2 .
   Solução:

     (a) Formamos o matriz com os vetores em colunas:
                               0                              1        0                                 1       0                                1
                                    1       1    3        0                   1     1        3       0                   1     1         3    0
                          B
                          B         1      −1   −1        1   C B
                                                              C B             0    −2       −4       1   C B
                                                                                                         C B             0    −2        −4    1   C
                                                                                                                                                  C
                        V=B         1       1    3        0   C∼B             0     0        0       0   C∼B             0     0         0    0   C
                          B                                   C B                                        C B                                      C
                          @         1      −1   −1        1   A @             0    −2       −4       1   A @             0     0         0    0   A
                                    1       1    3        0                   0     0        0       0                   0     0         0    0
         Dai segue: dim ger(v1 , v2 , v3 , v4 ) = dim ger(v1 , v2 ) = dim ger(v3 , v4 ) = 2, o que implica: ger(v1 , v2 , v3 , v4 ) =
         ger(v1 , v2 ) = ger(v3 , v4 ) = 2. A dimensão é o posto do matriz V, isto é 2.
     (b) Omitindo as linhas com somente zeros:
                           „            ˛                     «       „             ˛                «       „                 ˛         1
                                                                                                                                              «
                               1      1 ˛  3          0                   1       1 ˛ 3          0                   1       0 ˛ 1
                                        ˛                         ∼                 ˛                    ∼                     ˛         2
                               0     −2 ˛ −4          1                   0       1 ˛ 2     −1
                                                                                             2
                                                                                                                     0       1 ˛ 2      −1
                                                                                                                                         2

         Segue: v3 = v1 + 2v2 , e: v4 = 1 v1 − 1 v2 .
                                        2      2
     (c) Trocando o ordem dos vetores:
                          „               ˛               «        „                ˛   1        1
                                                                                                         «       „             ˛    1     1
                                                                                                                                              «
                                3       0 ˛ 1      1                       1      0 ˛                                1       0 ˛
                                          ˛                   ∼                     ˛   3        3           ∼                 ˛    3
                                                                                                                                    4
                                                                                                                                          3
                               −4       1 ˛ 0     −2                      −4      1 ˛   0    −2                      0       1 ˛    3
                                                                                                                                         −2
                                                                                                                                          3
                        1          4                 1            2
         Segue: v1 =      v
                        3 3
                              +      v ,
                                   3 4
                                           e: v2 =     v
                                                     3 3
                                                              −     v .
                                                                  3 4

2. 6 pts. Dado os vetores em relação ao base canônica, (e1 , e2 , e3 , e4 ), em R4 :

              v1 = (1, 1, 1, 1)T           v2 = (1, −1, 1, −1)T                   v3 = (1, 1, −1, −1)T                       v4 = (1, −1, −1, 1)T

     (a) Mostre que os vi ’s são mutualmente ortogonais, isto é: vi · vj = 0 por i = j.
     (b) Encontrar um base ortonormal de R4 , (d1 , d2 , d3 , d4 ), tal que: di = ci vi .




                                                                          10                                                                          Ditado.txt
Dr. Ole Peter Smith
                                          Instituto de Matemática e Estatística
                                             Universidade Federal de Goiás
                                ole@mat.ufg.br - http://www.ime.ufg.br/docentes/olepeter



     (c) Encontrar uma relação matricial entre os coordenados em relação aos ei ’s (antigos), xA , e os coordenados em
         relação aos di ’s (novos), xN :

                                                                xA = D xN
     (d) Encontrar uma relação matricial entre os coordenados novos, xN , e os coordenados antigos, xA :

                                                                xN = D xA

     (e) Justificar que vale: D = D−1 = DT = D.
         (f) Encontrar os coordenados dos vetores:

                                                w1 = (−1, 1, −1, 1)T      w2 = (1, 2, 3, 4)T

            em relação ao base novo, (d1 , d2 , d3 , d4 ).
   Solution:
   6 pts. Dado os vetores em relação ao base canônica, (e1 , e2 , e3 , e4 ), em R4 :

                v1 = (1, 1, 1, 1)T       v2 = (1, −1, 1, −1)T       v3 = (1, 1, −1, −1)T         v4 = (1, −1, −1, 1)T

     (a) Mostre que os vi ’s são mutualmente ortogonais, isto é: vi · vj = 0 por i = j.
     (b) Encontrar um base ortonormal de R4 , (d1 , d2 , d3 , d4 ), tal que: di = ci vi .
     (c) Encontrar uma relação matricial entre os coordenados em relação aos ei ’s (antigos), xA , e os coordenados em
         relação aos di ’s (novos), xN :

                                                                xA = D xN
     (d) Encontrar uma relação matricial entre os coordenados novos, xN , e os coordenados antigos, xA :

                                                                xN = D xA

     (e) Justificar que vale: D = D−1 = DT = D.
         (f) Encontrar os coordenados dos vetores:

                                                w1 = (−1, 1, −1, 1)T      w2 = (1, 2, 3, 4)T

            em relação ao base novo, (d1 , d2 , d3 , d4 ).
   Solução:

     (a) Verificamos:

                     v1 · v2 = 1 − 1 + 1 − 1 = 0         v1 · v3 = 1 + 1 − 1 − 1 = 0        v1 · v4 = 1 − 1 − 1 + 1 = 0
                                     v2 · v3 = 1 − 1 − 1 + 1 = 0       v2 · v4 = 1 + 1 − 1 − 1 = 0
                                                       v3 · v4 = 1 − 1 + 1 − 1 = 0
                 1
            QED .
                                                                    √                   √
     (b) Normalizamos os vi ’s: |v1 | = |v2 | = |v3 | = |v4 | =      12 + 12 + 12 + 12 = 4 = 2. Assim:
                                                                    „              «T
                                                             v1       1 1    1 1
                                                        d1 =      =    , ,    ,
                                                            |v1 |     2 2    2 2
                                                                  „                    «T
                                                           v        1   1    1    1
                                                      d2 = 2 =        ,− ,     ,−
                                                          |v2 |     2   2    2    2
1 Quod   Erat Demonstrandum


                                                               11                                                   Ditado.txt
Dr. Ole Peter Smith
                                       Instituto de Matemática e Estatística
                                          Universidade Federal de Goiás
                           ole@mat.ufg.br - http://www.ime.ufg.br/docentes/olepeter


                                                                   „                  «T
                                                         v3            1 1  1   1
                                                 d3 =         =         , ,− ,−
                                                        |v3 |          2 2  2   2
                                                                   „                  «T
                                                         v4            1   1  1 1
                                                 d4 =         =          ,− ,− ,
                                                        |v4 |          2   2  2 2
        É claro que di ’s são ortonormais, pois: di · dj = 0 por i = j, e: di · di = 1.
    (c) Organizamos os di ’s como colunas em uma matriz:
                                                             1     1      1       1
                                                     0                                1      0                   1
                      0                          1                                          1        1    1    1
                        |        |      |      |             2
                                                             1
                                                                   2      2       2
                                                     B
                                                             2
                                                                  −1
                                                                   2
                                                                          1
                                                                          2
                                                                                 −1
                                                                                  2   C= 1B 1
                                                                                      C   B         −1    1   −1 C
                 D = @ d1       d2     d3     d4 A = B       1     1
                                                                                                                 C
                                                     @                   −1      −1   A  2@ 1        1   −1   −1 A
                        |        |      |      |             2
                                                             1
                                                                   2      2       2
                                                             2
                                                                  −1
                                                                   2
                                                                         −1
                                                                          2
                                                                                  1
                                                                                  2
                                                                                            1       −1   −1    1
        Temos:
                                                         0                           1
                                                        1           1      1       1
                                                    1B 1           −1      1      −1 C
                                                xA = @B                              Cx
                                                    2   1           1     −1      −1 A N
                                                        1          −1     −1       1
    (d) Por D ser ortogonal, temos: D−1 = DT . Observamos que D ainda é simétrica: D = DT , ou seja: D−1 = D.
        Assim, invertemos a equação acima:
                                                         0                           1
                                                          1         1      1       1
                                                      1B 1         −1      1      −1 C
                                                xN   = @B                            Cx
                                                      2   1         1     −1      −1 A A
                                                          1        −1     −1       1

    (e) Já justificamos isto no item anterior; por D ser ortogonal e simétrica, vale: D = D−1 = DT = D.
     (f) Encontramos w1 em coordenados novos:
                          0                        10             1        0             1 0      1
                          1        1      1      1     −1                 −1 + 1 − 1 + 1        0
                       1B 1
                        B         −1      1     −1 C B 1
                                                   CB             C = 1 B −1 − 1 − 1 − 1 C = B −2 C
                                                                  C     B                C B      C
                       2@ 1        1     −1     −1 A @ −1         A   2 @ −1 + 1 + 1 − 1 A @ 0 A
                          1       −1     −1      1      1                 −1 − 1 + 1 − 1        0
        Ilustrando que: w1 = −2d2 .
        Para w2 :
                            0                   10                 1       0               1 0            1
                             1   1    1    1                     1           1+2+3+4              5
                       1 B 1 −1       1 −1 C B                     C= 1
                                                                 2 C       B 1 − 2 + 3 − 4 C B −1         C
                                                                           @ 1 + 2 − 3 − 4 A = @ −2
                          B                     CB                         B               C B            C
                       2  @ 1    1 −1 −1 A @                     3 A  2                                   A
                             1 −1 −1       1                     4           1−2−3+4              0
        Podemos verificar: w2 = 5d1 − d2 − 2d3 .
3. 2 pts. (Cabeludo?) Ortogonalização de Graham-Schmidt
   Dado os vetores em R3 :
                              v1 = (1, 1, 1)T        v2 = (1, −1, 1)T            v3 = (1, 1, −1)T
    (a) Mostre que v1 , v2 , v3 não são mutualmente ortogonais.
    (b) Mostre que v1 , v2 , v3 são linearmente independentes.
    (c) Escolhendo: d1 = v1 e d2 = v2 + αd1 , mostre que por:

                                                                       d1 · v2
                                                             α=−
                                                                       d1 · d1
        obtemos um vetor, d2 ⊥ d1 . Encontrar d2 .


                                                             12                                                 Ditado.txt
Dr. Ole Peter Smith
                                     Instituto de Matemática e Estatística
                                        Universidade Federal de Goiás
                           ole@mat.ufg.br - http://www.ime.ufg.br/docentes/olepeter



 (d) Escolhendo: d3 = v3 + βd1 + γd2 , mostre que por:

                                                     d1 · v 3                d2 · v 3
                                               β=−                   γ=−
                                                     d1 · d1                 d2 · d2
       obtemos um vetor, d3 ⊥ d1 e d3 ⊥ d2 . Encontrar d3 .
Solution:
2 pts. (Cabeludo?) Ortogonalização de Graham-Schmidt
Dado os vetores em R3 :

                             v1 = (1, 1, 1)T     v2 = (1, −1, 1)T         v3 = (1, 1, −1)T

 (a) Mostre que v1 , v2 , v3 não são mutualmente ortogonais.
 (b) Mostre que v1 , v2 , v3 são linearmente independentes.
 (c) Escolhendo: d1 = v1 e d2 = v2 + αd1 , mostre que por:

                                                                d1 · v2
                                                         α=−
                                                                d1 · d1
       obtemos um vetor, d2 ⊥ d1 . Encontrar d2 .
 (d) Escolhendo: d3 = v3 + βd1 + γd2 , mostre que por:

                                                     d1 · v 3                d2 · v 3
                                               β=−                   γ=−
                                                     d1 · d1                 d2 · d2
       obtemos um vetor, d3 ⊥ d1 e d3 ⊥ d2 . Encontrar d3 .
Solução:

 (a) Os vi ’s não são mutualmente ortogonais, pois:

                                v1 · v2 = 1 − 1 + 1 = 1 = 0     v1 · v3 = 1 + 1 − 1 = 1 = 0


                                               v2 · v3 = 1 − 1 − 1 = −1 = 0
 (b)                             0                1 0                      1 0                   1
                                  |       |     |       1        1       1     1          1    1
                           V = @ v1      v2    v3 A = @ 1       −1       1 A∼@ 0         −2    0 A
                                  |       |     |       1        1      −1     0          0   −2
       Segue, que ρV = 3, ou seja: os vi ’s são linearmente independentes.
 (c) Na equação d2 = v2 + αd1 , fazemos o produto escalar com d1 :

                                                d1 · d2 = d1 · v2 + α(d1 · d1 )
       Exigindo d1 ⊥ d2 - ou seja: d1 · d2 = 0 - segue:

                                                                d1 · v2
                                                         α=−
                                                                d1 · d1
       Usando resultados anteriores:

                                                          d1 · v2 = 1
                                                          d1 · d1 = 3
       Assim: α =   −1,
                     3
                          e assim:




                                                        13                                           Ditado.txt
Dr. Ole Peter Smith
                                 Instituto de Matemática e Estatística
                                    Universidade Federal de Goiás
                       ole@mat.ufg.br - http://www.ime.ufg.br/docentes/olepeter


                                           0   1    0   1 0 2 1
                                             1        1        3
                                                  1
                                     d2 = @ −1 A − @ 1 A = @ − 4 A
                                                               3
                                                  3            2
                                             1        1        3

   Observamos:
                                           0     1 0 2 1
                                               1         3    2 4 2
                                   d1 · d2 = @ 1 A · @ − 4 A = − + = 0
                                                         3
                                                         2    3 3 3
                                               1         3
   Isto é: d2 ⊥ d1 .
(d) Na d3 = v3 + βd1 + γd2 fazemos o produto escalar com d1 e utilizamos que d1 · d2 = 0:

                                         d1 · d3 = d1 · v3 + β(d1 · d1 ) + 0
   Exigindo d1 · d3 = 0, obtemos:

                                                          d1 · v3
                                                   β=−
                                                          d1 · d1
   Fazendo o produto escalar com d2 , segue de mesmo maneira:

                                                          d2 · v 3
                                                   γ=−
                                                          d2 · d2
   Usando resultados anteriores:

                                                    d1 · v3 = 1
                                                    d1 · d1 = 3
   Assim:
                                                              1
                                                       β=−
                                                              3
   E mais:

                                                                  4
                                                   d2 · v 3 = −
                                                                  3
                                                             24   8
                                                 d2 · d2 =      =
                                                              9   3
   Assim:
                                                             1
                                                       γ=
                                                             2
   Finalmente calculamos d3 :
                                    0 1     0   1    0 2 1 0    1
                                    1         1               1
                                          1@       1@ 3 A @
                             d3 = @ 1 A −     1 A+    −4
                                                       3
                                                          =   0 A
                                          3        2   2
                                   −1         1        3
                                                             −1

   Observamos:
                                             0    1 0             1
                                                1         1
                                    d1 · d3 = @ 1 A · @ 0         A=1+0−1=0
                                                1      −1
                                             0    1 0 2          1
                                               1         3
                                   d2 · d3 = @ 1 A · @ − 4       A= 2 +0− 2 =0
                                                         3
                                                         2          3     3
                                               1         3

   Isto é: d3 ⊥ d1 e d3 ⊥ d2 .



                                                  14                                        Ditado.txt
Provas.gabaritos.2010
Provas.gabaritos.2010
Provas.gabaritos.2010
Provas.gabaritos.2010
Provas.gabaritos.2010
Provas.gabaritos.2010
Provas.gabaritos.2010
Provas.gabaritos.2010
Provas.gabaritos.2010
Provas.gabaritos.2010
Provas.gabaritos.2010
Provas.gabaritos.2010
Provas.gabaritos.2010
Provas.gabaritos.2010
Provas.gabaritos.2010
Provas.gabaritos.2010
Provas.gabaritos.2010
Provas.gabaritos.2010
Provas.gabaritos.2010
Provas.gabaritos.2010
Provas.gabaritos.2010
Provas.gabaritos.2010
Provas.gabaritos.2010
Provas.gabaritos.2010
Provas.gabaritos.2010
Provas.gabaritos.2010
Provas.gabaritos.2010
Provas.gabaritos.2010
Provas.gabaritos.2010
Provas.gabaritos.2010
Provas.gabaritos.2010
Provas.gabaritos.2010
Provas.gabaritos.2010
Provas.gabaritos.2010
Provas.gabaritos.2010
Provas.gabaritos.2010
Provas.gabaritos.2010
Provas.gabaritos.2010
Provas.gabaritos.2010
Provas.gabaritos.2010
Provas.gabaritos.2010
Provas.gabaritos.2010
Provas.gabaritos.2010
Provas.gabaritos.2010
Provas.gabaritos.2010
Provas.gabaritos.2010
Provas.gabaritos.2010
Provas.gabaritos.2010
Provas.gabaritos.2010
Provas.gabaritos.2010
Provas.gabaritos.2010
Provas.gabaritos.2010
Provas.gabaritos.2010
Provas.gabaritos.2010
Provas.gabaritos.2010
Provas.gabaritos.2010
Provas.gabaritos.2010
Provas.gabaritos.2010
Provas.gabaritos.2010
Provas.gabaritos.2010
Provas.gabaritos.2010
Provas.gabaritos.2010
Provas.gabaritos.2010
Provas.gabaritos.2010
Provas.gabaritos.2010
Provas.gabaritos.2010
Provas.gabaritos.2010
Provas.gabaritos.2010
Provas.gabaritos.2010
Provas.gabaritos.2010
Provas.gabaritos.2010
Provas.gabaritos.2010
Provas.gabaritos.2010
Provas.gabaritos.2010
Provas.gabaritos.2010
Provas.gabaritos.2010
Provas.gabaritos.2010

Mais conteúdo relacionado

Mais procurados

Fisica exercicios resolvidos 007
Fisica exercicios resolvidos  007Fisica exercicios resolvidos  007
Fisica exercicios resolvidos 007comentada
 
Gab complexos formaalgebrica2012
Gab complexos formaalgebrica2012Gab complexos formaalgebrica2012
Gab complexos formaalgebrica2012Wilson Marques
 
Mat utfrs 03. potenciacao
Mat utfrs 03. potenciacaoMat utfrs 03. potenciacao
Mat utfrs 03. potenciacaotrigono_metria
 
Gabarito do 2º teste de geometria
Gabarito do 2º teste de geometriaGabarito do 2º teste de geometria
Gabarito do 2º teste de geometriaProfessor Carlinhos
 
2010 volume2 cadernodoaluno_matematica_ensinofundamentalii_8aserie_gabarito
2010 volume2 cadernodoaluno_matematica_ensinofundamentalii_8aserie_gabarito2010 volume2 cadernodoaluno_matematica_ensinofundamentalii_8aserie_gabarito
2010 volume2 cadernodoaluno_matematica_ensinofundamentalii_8aserie_gabaritoprofzwipp
 
Mat utfrs 05. radiciacao
Mat utfrs 05. radiciacaoMat utfrs 05. radiciacao
Mat utfrs 05. radiciacaotrigono_metria
 
Matrizes e determinantes res
Matrizes e determinantes resMatrizes e determinantes res
Matrizes e determinantes resIsabella Silva
 
Mat 140 questoes resolvidas vol i
Mat 140 questoes resolvidas vol iMat 140 questoes resolvidas vol i
Mat 140 questoes resolvidas vol itrigono_metrico
 
Matemática provas de vestibulares ita 1.101 questões + gabaritos
Matemática provas de vestibulares ita  1.101 questões + gabaritosMatemática provas de vestibulares ita  1.101 questões + gabaritos
Matemática provas de vestibulares ita 1.101 questões + gabaritosprof. Renan Viana
 

Mais procurados (11)

Fisica exercicios resolvidos 007
Fisica exercicios resolvidos  007Fisica exercicios resolvidos  007
Fisica exercicios resolvidos 007
 
Gab complexos formaalgebrica2012
Gab complexos formaalgebrica2012Gab complexos formaalgebrica2012
Gab complexos formaalgebrica2012
 
Mat utfrs 03. potenciacao
Mat utfrs 03. potenciacaoMat utfrs 03. potenciacao
Mat utfrs 03. potenciacao
 
Gabarito do 2º teste de geometria
Gabarito do 2º teste de geometriaGabarito do 2º teste de geometria
Gabarito do 2º teste de geometria
 
2010 volume2 cadernodoaluno_matematica_ensinofundamentalii_8aserie_gabarito
2010 volume2 cadernodoaluno_matematica_ensinofundamentalii_8aserie_gabarito2010 volume2 cadernodoaluno_matematica_ensinofundamentalii_8aserie_gabarito
2010 volume2 cadernodoaluno_matematica_ensinofundamentalii_8aserie_gabarito
 
Mat utfrs 05. radiciacao
Mat utfrs 05. radiciacaoMat utfrs 05. radiciacao
Mat utfrs 05. radiciacao
 
Matrizes e determinantes res
Matrizes e determinantes resMatrizes e determinantes res
Matrizes e determinantes res
 
Mat 140 questoes resolvidas vol i
Mat 140 questoes resolvidas vol iMat 140 questoes resolvidas vol i
Mat 140 questoes resolvidas vol i
 
Matemática provas de vestibulares ita 1.101 questões + gabaritos
Matemática provas de vestibulares ita  1.101 questões + gabaritosMatemática provas de vestibulares ita  1.101 questões + gabaritos
Matemática provas de vestibulares ita 1.101 questões + gabaritos
 
Aulas Cap 6
Aulas Cap 6Aulas Cap 6
Aulas Cap 6
 
Aulas Cap 4
Aulas Cap 4Aulas Cap 4
Aulas Cap 4
 

Semelhante a Provas.gabaritos.2010

Mat matrizes determinantes 001 exercicios
Mat matrizes determinantes  001 exerciciosMat matrizes determinantes  001 exercicios
Mat matrizes determinantes 001 exerciciostrigono_metrico
 
Determinantes sistemas lineares [modo de compatibilidade]
Determinantes sistemas lineares [modo de compatibilidade]Determinantes sistemas lineares [modo de compatibilidade]
Determinantes sistemas lineares [modo de compatibilidade]AUTONOMO
 

Semelhante a Provas.gabaritos.2010 (6)

Mat matrizes determinantes 001 exercicios
Mat matrizes determinantes  001 exerciciosMat matrizes determinantes  001 exercicios
Mat matrizes determinantes 001 exercicios
 
Ap mat potenciacao
Ap mat potenciacaoAp mat potenciacao
Ap mat potenciacao
 
Matriz
MatrizMatriz
Matriz
 
Determinantes sistemas lineares [modo de compatibilidade]
Determinantes sistemas lineares [modo de compatibilidade]Determinantes sistemas lineares [modo de compatibilidade]
Determinantes sistemas lineares [modo de compatibilidade]
 
Matrizes e determinantes
Matrizes e determinantesMatrizes e determinantes
Matrizes e determinantes
 
1ano Ufsmat2005
1ano Ufsmat20051ano Ufsmat2005
1ano Ufsmat2005
 

Mais de Ole Peter Smith (18)

Dia.debian 2013, Trinidade-GO
Dia.debian 2013, Trinidade-GODia.debian 2013, Trinidade-GO
Dia.debian 2013, Trinidade-GO
 
Provas Algebra Linear, Gabaritos
Provas Algebra Linear, GabaritosProvas Algebra Linear, Gabaritos
Provas Algebra Linear, Gabaritos
 
Provas Algebra Linear
Provas Algebra LinearProvas Algebra Linear
Provas Algebra Linear
 
Formas quadraticas
Formas quadraticasFormas quadraticas
Formas quadraticas
 
Notas.al
Notas.alNotas.al
Notas.al
 
Compendio.algebra.linear
Compendio.algebra.linearCompendio.algebra.linear
Compendio.algebra.linear
 
Januaria
JanuariaJanuaria
Januaria
 
Wi.ufpr
Wi.ufprWi.ufpr
Wi.ufpr
 
Tcp50.jatai
Tcp50.jataiTcp50.jatai
Tcp50.jatai
 
Tcp50.jatai.2
Tcp50.jatai.2Tcp50.jatai.2
Tcp50.jatai.2
 
Unesp
UnespUnesp
Unesp
 
Fisl
FislFisl
Fisl
 
Gc.d day
Gc.d dayGc.d day
Gc.d day
 
Ditados
DitadosDitados
Ditados
 
Frel.vv
Frel.vvFrel.vv
Frel.vv
 
Goias
GoiasGoias
Goias
 
Provas.2010
Provas.2010Provas.2010
Provas.2010
 
Frel.vv
Frel.vvFrel.vv
Frel.vv
 

Provas.gabaritos.2010

  • 1. Dr. Ole Peter Smith Instituto de Matemática e Estatística Universidade Federal de Goiás ole@mat.ufg.br - http://www.ime.ufg.br/docentes/olepeter Data: 15/04/2009 Semestre: Curso: Engenharia de Computação Disciplina: Álgebra Linear Prova: I 1. 4 pts. Dado as matrizes: 0 1 0 1 1 −2 0 −1 0 0 A=@ 0 1 1 A B=@ 0 2 0 A 1 −2 1 0 0 3 (a) Encontrar: det A e det B. (b) Encontrar as matrizes adjuntas: A∗ e B ∗ . (c) Encontrar os inversos: A−1 e B −1 . (d) Encontrar os determinantes dos inversos: det (A−1 ) e det (B −1 ). (e) Encontrar os produtos: A B e B A. (f) Mostre que em geral vale por matrizes do mesmo ordem: (A B)−1 = B −1 A−1 . (g) Encontrar o inverso do produto: (A B)−1 . (h) Encontrar o inverso do produto: (B A)−1 . Solution: (a) ˛ ˛ ˛ ˛ ˛ 1 ˛ −2 0 ˛ ˛ 1 −2 0 ˛ ˛ ˛ ˛ det A = ˛ 0 ˛ 1 1 ˛=˛ 0 ˛ ˛ 1 1 ˛=1·1·1=1 ˛ ˛ 1 −2 1 ˛ ˛ 0 0 1 ˛ ˛ ˛ ˛ −1 0 0 ˛ ˛ ˛ det B = ˛ 0 ˛ 2 0 ˛ = (−1) · 2 · 3 = −6 ˛ ˛ 0 0 3 ˛ (b) Para A: ˛ ˛ ˛ ˛ ˛ ˛ ˛ 1 1 ˛ ˛ 0 1 ˛ ˛ 0 1 ˛ D11 = ˛ ˛ −2 1 ˛ = 3 ˛ 1 1 ˛ = −1 D12 = ˛ ˛ 1 −2 ˛ = −1 D13 = ˛ ˛ ˛ ˛ ˛ ˛ ˛ ˛ ˛ ˛ ˛ −2 0 ˛ ˛ 1 0 ˛ ˛ 1 −2 ˛ D21 = ˛ ˛ −2 1 ˛ = −2 D22 = ˛˛ 1 1 ˛=1 D23 = ˛ ˛ 1 −2 ˛ = 0 ˛ ˛ ˛ ˛ ˛ ˛ ˛ ˛ ˛ ˛ −2 0 ˛ ˛ 1 0 ˛ ˛ 1 −2 ˛ D31 = ˛ ˛ ˛ = −2 D32 = ˛˛ ˛=1 D33 = ˛ ˛ ˛=1 1 1 ˛ 0 1 ˛ 0 1 ˛ Assim: A11 = 3 A12 = 1 A13 = −1 A21 = 2 A22 = 1 A23 = 0 A31 = −2 A32 = −1 A33 = 1 Ou seja: 0 1 3 2 −2 A∗ = (Aji ) = @ 1 1 −1 A −1 0 1 Por B ser diagonal: Dij = Aij = 0, i = j 1 Quem entende sério somente por sério E brincadeira somente por brincadeira De fato desentendeu ambos... Piet Hein
  • 2. Dr. Ole Peter Smith Instituto de Matemática e Estatística Universidade Federal de Goiás ole@mat.ufg.br - http://www.ime.ufg.br/docentes/olepeter ˛ ˛ ˛ ˛ ˛ ˛ ˛ 2 0 ˛ ˛ −1 0 ˛ ˛ −1 0 ˛ D11 = A11 =˛ ˛=6 D22 = A22 =˛ ˛ = −3 D33 = A33 =˛ ˛ = −2 ˛ 0 3 ˛ ˛ 0 3 ˛ ˛ 0 2 ˛ Ou seja: 0 1 6 0 0 ∗ B =@ 0 −3 0 A 0 0 −2 (c) Encontrar os inversos: A−1 e B −1 . 0 1 3 2 −2 −1 1 ∗ A = A =@ 1 1 −1 A det A −1 0 1 E: 0 1 −1 0 0 −1 1 ∗ 1 B = B =@ 0 2 0 A det B 1 0 0 3 (d) 1 det (A−1 ) = =1 det A 1 1 det (B −1 ) = =− det B 6 (e) 0 10 1 0 1 1 −2 0 −1 0 0 1 −4 0 AB =@ 0 1 1 A@ 0 2 0 A=@ 0 2 3 A 1 −2 1 0 0 3 −1 −4 3 0 10 1 0 1 −1 0 0 1 −2 0 −1 2 0 BA=@ 0 2 0 A@ 0 1 1 A=@ 0 2 2 A 0 0 3 1 −2 1 3 −6 3 (f) Por o inverso ser único e o produto de matrizes ser associativo: (B −1 A−1 )(A B) = B −1 (A−1 A)B = B −1 I B = B −1 B = I E: (A B) (B −1 A−1 ) = A(B B −1 )A−1 = A I A−1 = A A−1 = I QED (g) 0 10 1 0 1 −1 0 0 3 2 −2 −3 −2 2 −1 −1 −1 1 (A B) =B A =@ 0 2 0 A@ 1 1 −1 A = @ 12 1 2 −21 A 1 0 0 3 −1 0 1 −13 0 1 3 (h) Encontrar o inverso do produto: (B A)−1 . −2 0 10 1 0 1 3 2 −2 −1 0 0 −3 1 3 −1 −1 −1 1 1 1 A (B A) =A B =@ 1 1 −1 A @ 0 2 0 A = @ −1 2 −3 1 1 −1 0 1 0 0 3 1 0 3 2. 4 pts. Dado a sistema linear: 8 9 < x1 + 2x2 + 3x3 + 4x4 + 5x5 = 1 = (∗) : 2x1 + 3x2 + 4x3 + 5x4 − x5 = 2 3x1 + 4x2 + 5x3 + 6x4 − 3x5 = 3 : ; 2 Quem entende sério somente por sério E brincadeira somente por brincadeira De fato desentendeu ambos... Piet Hein
  • 3. Dr. Ole Peter Smith Instituto de Matemática e Estatística Universidade Federal de Goiás ole@mat.ufg.br - http://www.ime.ufg.br/docentes/olepeter (a) Encontrar a solução completa do sistema homogêneo do (∗). (b) Encontrar a solução completa do sistema não homogêneo. (c) Encontrar a solução completa do sistema: 8 9 < x1 + 2x2 + 3x3 + 4x4 + 5x5 = 2 = (∗∗) : 2x1 + 3x2 + 4x3 + 5x4 − x5 = 3 3x1 + 4x2 + 5x3 + 6x4 − 3x5 = 4 : ; Solution: Resolvemos tudo de uma vez: 0 1 1 2 3 4 5 | 1 2 @ 2 3 4 5 −1 | 2 3 A∼ 3 4 5 6 −3 | 3 4 0 1 1 2 3 4 5 | 1 2 @ 0 −1 −2 −3 −11 | 0 −1 A ∼ 0 −2 −4 −6 −18 | 0 −2 0 1 1 0 −1 −2 −17 | 1 0 @ 0 1 2 3 11 | 0 1 A∼ 0 0 0 0 −4 | 0 0 0 1 1 0 −1 −2 0 | 1 0 @ 0 1 2 3 0 | 0 1 A 0 0 0 0 1 | 0 0 Pondo x3 = t e x4 = s obtemos a solução completa do sistema homogêneo: (x1 , x2 , x3 , x4 , x5 )T = (t + 2s, −2t − 3s, t, s, 0)T , (t, s) ∈ R2 E por (∗): (x1 , x2 , x3 , x4 , x5 )T = (1 + t + 2s, −2t − 3s, t, s, 0)T , (t, s) ∈ R2 E por (∗∗): (x1 , x2 , x3 , x4 , x5 )T = (t + 2s, 1 − 2t − 3s, t, s, 0)T , (t, s) ∈ R2 Comentário: Verificar: (i) Verifique que a solução da eq. homogênea satisfaz as eqs. (ii) Verifique que as soluções particulares satisfaz (∗) resp. (∗∗). 3. 2 pts. (Cabeludo) Dado as matrizes: 0 1 1 2 3 4 B 4 3 2 1 C A=B C , (a, b) ∈ R2 @ a 2 3 4 A 4 3 2 b 0 1 0 a 0 0 B a 0 a 0 C B=@ B C, a ∈ R 0 a 0 a A 0 0 a 0 (a) Encontrar para quaisquer valores de a e b o determinante do A. (b) Encontrar para quaisquer valores de a e b o posto do A. (c) Encontrar para qualquer valor de a o determinante do B. 3 Quem entende sério somente por sério E brincadeira somente por brincadeira De fato desentendeu ambos... Piet Hein
  • 4. Dr. Ole Peter Smith Instituto de Matemática e Estatística Universidade Federal de Goiás ole@mat.ufg.br - http://www.ime.ufg.br/docentes/olepeter (d) Para quais valores de a o matriz B tem inverso? Para estes valores, encontrar o inverso. Solution: (a) ˛ ˛ ˛ ˛ ˛ ˛ 1 2 3 4 ˛ ˛ 1 ˛ ˛ 2 3 4 ˛ ˛ ˛ 4 3 2 1 ˛ ˛ 4 3 2 1 ˛ det A = ˛ ˛=˛ ˛ ˛ a−1 0 0 ˛= ˛ ˛ a 2 3 4 ˛ ˛ 0 ˛ ˛ ˛ 4 3 2 b ˛ ˛ 0 0 0 b−1 ˛ ˛ ˛ 3+1 4+4 ˛ 2 3 ˛ (−1) (−1) (a − 1)(b − 1) ˛ ˛ 3 ˛ = (a − 1)(b − 1)(4 − 9) = −5(a − 1)(b − 1) 2 ˛ (b) Como no item anterior: 0 1 0 1 1 2 3 4 1 2 3 4 B 4 3 2 1 C B 4 3 2 1 C B C∼B C @ a 2 3 4 A @ a−1 0 0 0 A 4 3 2 b 0 0 0 b−1 Donde segue: 8 > 4, > a=1∧b=1 3, a=1∧b=1 < ρA = > 3, > a=1∧b=1 2, a=b=1 : (c) ˛ ˛ ˛ ˛ ˛ ˛ ˛ ˛ ˛ ˛ ˛ ˛ 0 a 0 0 ˛ ˛ ˛ ˛ 0 1 0 0 ˛ ˛ ˛ ˛ 0 1 0 0 ˛ ˛ ˛ ˛ 1 0 0 0 ˛ ˛ ˛ ˛ 1 0 0 0 ˛ ˛ ˛ a 0 a 0 ˛ 4˛ ˛ 1 0 1 0 ˛ 4˛ ˛ 1 0 0 0 ˛ 4˛ ˛ 0 1 0 0 ˛ 4˛ ˛ 0 1 0 0 ˛ = a4 ˛ det B = ˛ ˛=a ˛=a ˛=a ˛=a ˛ ˛ 0 a 0 a ˛ ˛ ˛ ˛ 0 1 0 1 ˛ ˛ ˛ ˛ 0 0 0 1 ˛ ˛ ˛ ˛ 0 0 0 1 ˛ ˛ ˛ ˛ 0 0 1 0 ˛ ˛ ˛ 0 0 a 0 ˛ ˛ 0 0 1 0 ˛ ˛ 0 0 1 0 ˛ ˛ 0 0 1 0 ˛ ˛ 0 0 0 1 ˛ (d) O inverso existe se e somente se o determinante não for zero: a4 = 0 ⇔ a = 0. Por a = 0 obtemos: 0 1 0 1 0 a 0 0 | 1 0 0 0 0 a 0 0 | 1 0 0 0 B a 0 a 0 | 0 1 0 0 C B a 0 0 0 | 0 1 0 −1 C @ 0 a 0 a | 0 0 1 0 A∼@ 0 B C B C∼ 0 0 a | −1 0 1 0 A 0 0 a 0 | 0 0 0 1 0 0 a 0 | 0 0 0 1 1 1 0 1 0 1 a 0 0 0 | 0 1 0 −1 1 0 0 0 | 0 a 0 −a 1 B 0 B a 0 0 | 1 0 0 0 C B 0 C∼B 1 0 0 | a 0 0 0 CC 1 @ 0 0 a 0 | 0 0 0 1 A @ 0 0 1 0 | 0 0 0 a A 1 1 0 0 0 a | −1 0 1 0 0 0 0 1 | −a 0 a 0 Assim: 1 1 0 1 0 a 0 −a 1 0 0 0 C B −1 = B B a C 1 @ 0 0 0 a A 1 1 −a 0 a 0 4 Quem entende sério somente por sério E brincadeira somente por brincadeira De fato desentendeu ambos... Piet Hein
  • 5. Dr. Ole Peter Smith Instituto de Matemática e Estatística Universidade Federal de Goiás ole@mat.ufg.br - http://www.ime.ufg.br/docentes/olepeter Data: 05/05/2009 Semestre: Curso: Engenharia Civil Disciplina: Álgebra Linear Prova: I 1. 4 pts. Dado as matrizes: 0 1 0 1 1 −2 0 −1 0 0 A=@ 0 1 1 A B=@ 0 2 0 A 1 −2 1 0 0 3 Sabendo que: 1 det (A−1 ) = det A−1 = det A e: (A B)−1 = B −1 A−1 responde o seguinte: (a) Encontrar: det A e det B. (b) Encontrar as matrizes adjuntas: A∗ e B ∗ . (c) Encontrar os inversos: A−1 e B −1 . (d) Encontrar os determinantes dos inversos: det (A−1 ) e det (B −1 ). (e) Encontrar os produtos: A B e B A. (f) Encontrar o inverso do produto: (A B)−1 . Solution: 4 pts. Dado as matrizes: 0 1 0 1 1 −2 0 −1 0 0 A=@ 0 1 1 A B=@ 0 2 0 A 1 −2 1 0 0 3 Sabendo que: 1 det (A−1 ) = det A−1 = det A e: (A B)−1 = B −1 A−1 responde o seguinte: (a) Encontrar: det A e det B. ˛ ˛ ˛ ˛ ˛ 1 ˛ −2 0 ˛ ˛ 1 −2 0 ˛ ˛ ˛ ˛ det A = ˛ 0 ˛ 1 1 ˛=˛ 0 ˛ ˛ 1 1 ˛=1·1·1=1 ˛ ˛ 1 −2 1 ˛ ˛ 0 0 1 ˛ ˛ ˛ ˛ −1 0 0 ˛ ˛ ˛ det B = ˛ 0 ˛ 2 0 ˛ = (−1) · 2 · 3 = −6 ˛ ˛ 0 0 3 ˛ 5 I really do like Your Christ But I do dislike Your Christians Why can’t Your Christians be more like Your Christ? - Mahatma Gandhi
  • 6. Dr. Ole Peter Smith Instituto de Matemática e Estatística Universidade Federal de Goiás ole@mat.ufg.br - http://www.ime.ufg.br/docentes/olepeter (b) Encontrar as matrizes adjuntas: A∗ e B ∗ . Para A: ˛ ˛ ˛ ˛ ˛ ˛ ˛ 1 1 ˛ ˛ 0 1 ˛ ˛ 0 1 ˛ D11 = ˛ ˛ ˛=3 D12 = ˛ ˛ = −1 D13 = ˛ ˛ ˛ = −1 −2 1 ˛ ˛ 1 1 ˛ 1 −2 ˛ ˛ ˛ ˛ ˛ ˛ ˛ ˛ −2 0 ˛ ˛ 1 0 ˛ ˛ 1 −2 ˛ D21 = ˛ ˛ −2 1 ˛ = −2 D22 = ˛ ˛=1 D23 = ˛ ˛ 1 −2 ˛ = 0 ˛ ˛ ˛ 1 1 ˛ ˛ ˛ ˛ ˛ ˛ ˛ ˛ −2 0 ˛ ˛ 1 0 ˛ ˛ 1 −2 ˛ D31 = ˛ ˛ ˛ = −2 D32 = ˛ ˛=1 D33 = ˛ ˛ ˛=1 1 1 ˛ ˛ 0 1 ˛ 0 1 ˛ Assim: A11 = 3 A12 = 1 A13 = −1 A21 = 2 A22 = 1 A23 = 0 A31 = −2 A32 = −1 A33 = 1 Ou seja: 0 1 3 2 −2 A∗ = (Aji ) = @ 1 1 −1 A −1 0 1 Por B ser diagonal: Dij = Aij = 0, i = j ˛ ˛ ˛ ˛ ˛ ˛ ˛ 2 0 ˛ ˛ −1 0 ˛ ˛ −1 0 ˛ D11 = A11 = ˛ ˛=6 D22 = A22 = ˛ ˛ = −3 D33 = A33 = ˛ ˛ = −2 ˛ 0 3 ˛ ˛ 0 3 ˛ ˛ 0 2 ˛ Ou seja: 0 1 6 0 0 ∗ B =@ 0 −3 0 A 0 0 −2 (c) Encontrar os inversos: A−1 e B −1 . 0 1 3 2 −2 −1 1 ∗ A = A =@ 1 1 −1 A det A −1 0 1 E: 0 1 −1 0 0 1 B −1 = B∗ = @ 0 1 2 0 A det B 1 0 0 3 (d) Encontrar os determinantes dos inversos: det (A−1 ) e det (B −1 ). 1 det (A−1 ) = =1 det A 1 1 det (B −1 ) = =− det B 6 (e) Encontrar os produtos: A B e B A. 0 10 1 0 1 1 −2 0 −1 0 0 1 −4 0 AB =@ 0 1 1 A@ 0 2 0 A=@ 0 2 3 A 1 −2 1 0 0 3 −1 −4 3 0 10 1 0 1 −1 0 0 1 −2 0 −1 2 0 BA=@ 0 2 0 A@ 0 1 1 A=@ 0 2 2 A 0 0 3 1 −2 1 3 −6 3 6 I really do like Your Christ But I do dislike Your Christians Why can’t Your Christians be more like Your Christ? - Mahatma Gandhi
  • 7. Dr. Ole Peter Smith Instituto de Matemática e Estatística Universidade Federal de Goiás ole@mat.ufg.br - http://www.ime.ufg.br/docentes/olepeter (f) Encontrar o inverso do produto: (A B)−1 . 0 10 1 0 1 −1 0 0 3 2 −2 −3 −2 2 −1 −1 −1 1 (A B) =B A =@ 0 2 0 A@ 1 1 −1 A = @ 12 1 2 −21 A 1 0 0 3 −1 0 1 −13 0 1 3 2. 4 pts. Dado a sistema linear: 8 9 < x1 + 2x2 + 3x3 + 4x4 + 5x5 = 1 = (∗) : 2x1 + 3x2 + 4x3 + 5x4 − x5 = 2 3x1 + 4x2 + 5x3 + 6x4 − 3x5 = 3 : ; (a) Encontrar a solução completa do sistema homogêneo do (∗). (b) Encontrar a solução completa do sistema não homogêneo. (c) Encontrar a solução completa do sistema: 8 9 < x1 + 2x2 + 3x3 + 4x4 + 5x5 = 2 = (∗∗) : 2x1 + 3x2 + 4x3 + 5x4 − x5 = 3 3x1 + 4x2 + 5x3 + 6x4 − 3x5 = 4 : ; Solution: 4 pts. Dado a sistema linear: 8 9 < x1 + 2x2 + 3x3 + 4x4 + 5x5 = 1 = (∗) : 2x1 + 3x2 + 4x3 + 5x4 − x5 = 2 3x1 + 4x2 + 5x3 + 6x4 − 3x5 = 3 : ; (a) Encontrar a solução completa do sistema homogêneo do (∗). (b) Encontrar a solução completa do sistema não homogêneo. (c) Encontrar a solução completa do sistema: 8 9 < x1 + 2x2 + 3x3 + 4x4 + 5x5 = 2 = (∗∗) : 2x1 + 3x2 + 4x3 + 5x4 − x5 = 3 3x1 + 4x2 + 5x3 + 6x4 − 3x5 = 4 : ; Resolvemos tudo de uma vez: 0 ˛ 1 0 ˛ 1 1 2 3 4 5 ˛ 1 ˛ 2 1 2 3 4 5 ˛ 1 ˛ 2 @ 2 3 4 5 −1 ˛ 2 ˛ 3 A ∼ @ 0 −1 −2 −3 −11 ˛ 0 −1 ˛ A∼ 3 4 5 6 −3 ˛ 3 4 0 −2 −4 −6 −18 ˛ 0 −2 0 ˛ 1 0 ˛ 1 1 0 −1 −2 −17 ˛ ˛ 1 0 1 0 −1 −2 0 ˛ 1 0 ˛ @ 0 1 2 3 11 ˛ ˛ 0 1 A∼@ 0 1 2 3 0 ˛ 0 1 ˛ A 0 0 0 0 −4 ˛ 0 0 0 0 0 0 1 ˛ 0 0 7 I really do like Your Christ But I do dislike Your Christians Why can’t Your Christians be more like Your Christ? - Mahatma Gandhi
  • 8. Dr. Ole Peter Smith Instituto de Matemática e Estatística Universidade Federal de Goiás ole@mat.ufg.br - http://www.ime.ufg.br/docentes/olepeter Pondo x3 = t e x4 = s obtemos a solução completa do sistema homogêneo: 0 1 0 1 0 1 0 1 x1 1 2 t + 2s B x2 C B −2 C B −3 C B −2t − 3s C C , (t, s) ∈ R2 B C B C B C B C B B x3 C = tB C B 1 C + sB C B 0 C=B C B t C @ x4 A @ 0 A @ 1 A @ s A x5 0 0 0 E por (∗): 0 1 0 1 0 1 0 1 0 1 x1 1 1 2 1 + t + 2s B x2 C B 0 C B −2 C B −3 C B −2t − 3s C C , (t, s) ∈ R2 B C B C B C B C B C B B x3 C=B C B 0 C + tB C B 1 C + sB C B 0 C=B C B t C @ x4 A @ 0 A @ 0 A @ 1 A @ s A x5 0 0 0 0 E por (∗∗): 0 1 0 1 0 1 0 1 0 1 x1 0 1 2 t + 2s B x2 C B 1 C B −2 C B −3 C B 1 − 2t − 3s C C , (t, s) ∈ R2 B C B C B C B C B C B x3 C = B 0 C + tB 1 C + sB 0 C = B t B C B C B C B C B C @ x4 A @ 0 A @ 0 A @ 1 A @ s A x5 0 0 0 0 Comentário: Controle: (i) Verifique que a solução da eqação homogênea satisfaz as eqções. (ii) Verifique que as soluções particulares satisfaz (∗) resp. (∗∗). 3. 2 pts. (Cabeludo) Dado as matrizes: 0 1 1 2 3 4 B 4 3 2 1 C A=B @ a C, (a, b) ∈ R2 2 3 4 A 4 3 2 b 0 1 0 a 0 0 B a 0 a 0 C B=@ B C, a∈R 0 a 0 a A 0 0 a 0 (a) Encontrar para quaisquer valores de a e b o determinante do A. (b) Encontrar para quaisquer valores de a e b o posto do A. (c) Encontrar para qualquer valor de a o determinante do B. (d) Para quais valores de a o matriz B tem inverso? Para estes valores, encontrar o inverso. Solution: 2 pts. (Cabeludo) Dado as matrizes: 0 1 1 2 3 4 B 4 3 2 1 C A=B @ a C, (a, b) ∈ R2 2 3 4 A 4 3 2 b 0 1 0 a 0 0 B a 0 a 0 C B=@ B C, a∈R 0 a 0 a A 0 0 a 0 8 I really do like Your Christ But I do dislike Your Christians Why can’t Your Christians be more like Your Christ? - Mahatma Gandhi
  • 9. Dr. Ole Peter Smith Instituto de Matemática e Estatística Universidade Federal de Goiás ole@mat.ufg.br - http://www.ime.ufg.br/docentes/olepeter (a) Encontrar para quaisquer valores de a e b o determinante do A. ˛ ˛ ˛ ˛ ˛ ˛ 1 2 3 4 ˛ ˛ 1 ˛ ˛ 2 3 4 ˛ ˛ ˛ 4 3 2 1 ˛ ˛ 4 3 2 1 ˛ det A = ˛ ˛=˛ ˛ ˛ a−1 0 0 ˛= ˛ ˛ a 2 3 4 ˛ ˛ 0 ˛ ˛ ˛ 4 3 2 b ˛ ˛ 0 0 0 b−1 ˛ ˛ ˛ 3+1 4+4 ˛ 2 3 ˛ (−1) (−1) (a − 1)(b − 1) ˛ ˛ 3 ˛ = (a − 1)(b − 1)(4 − 9) = −5(a − 1)(b − 1) 2 ˛ (b) Encontrar para quaisquer valores de a e b o posto do A. Como no item anterior: 0 1 0 1 1 2 3 4 1 2 3 4 B 4 3 2 1 C B 4 3 2 1 C B C∼B C @ a 2 3 4 A @ a−1 0 0 0 A 4 3 2 b 0 0 0 b−1 Donde segue: 8 > 4, > a=1∧b=1 3, a=1∧b=1 < ρA = > 3, > a=1∧b=1 2, a=b=1 : (c) Encontrar para qualquer valor de a o determinante do B. ˛ ˛ ˛ ˛ ˛ ˛ ˛ ˛ ˛ ˛ ˛ ˛ 0 a 0 0 ˛ ˛ ˛ ˛ 0 1 0 0 ˛ ˛ ˛ ˛ 0 1 0 0 ˛ ˛ ˛ ˛ 1 0 0 0 ˛ ˛ ˛ ˛ 1 0 0 0 ˛ ˛ ˛ a 0 a 0 ˛ 4˛ ˛ 1 0 1 0 ˛ 4˛ ˛ 1 0 0 0 ˛ 4˛ ˛ 0 1 0 0 ˛ 4˛ ˛ 0 1 0 0 ˛ = a4 ˛ det B = ˛ ˛=a ˛=a ˛=a ˛=a ˛ ˛ 0 a 0 a ˛ ˛ ˛ ˛ 0 1 0 1 ˛ ˛ ˛ ˛ 0 0 0 1 ˛ ˛ ˛ ˛ 0 0 0 1 ˛ ˛ ˛ ˛ 0 0 1 0 ˛ ˛ ˛ 0 0 a 0 ˛ ˛ 0 0 1 0 ˛ ˛ 0 0 1 0 ˛ ˛ 0 0 1 0 ˛ ˛ 0 0 0 1 ˛ (d) Para quais valores de a o matriz B tem inverso? Para estes valores, encontrar o inverso. O inverso existe se e somente se o determinante não for zero: a4 = 0 ⇔ a = 0. Por a = 0 obtemos: 0 1 0 1 0 a 0 0 | 1 0 0 0 0 a 0 0 | 1 0 0 0 B a 0 a 0 | 0 1 0 0 C B a 0 0 0 | 0 1 0 −1 C @ 0 a 0 a | 0 0 1 0 A∼@ 0 B C B C∼ 0 0 a | −1 0 1 0 A 0 0 a 0 | 0 0 0 1 0 0 a 0 | 0 0 0 1 1 1 0 1 0 1 a 0 0 0 | 0 1 0 −1 1 0 0 0 | 0 a 0 −a 1 B 0 B a 0 0 | 1 0 0 0 C B 0 C∼B 1 0 0 | a 0 0 0 CC 1 @ 0 0 a 0 | 0 0 0 1 A @ 0 0 1 0 | 0 0 0 a A 1 1 0 0 0 a | −1 0 1 0 0 0 0 1 | −a 0 a 0 Assim: 1 1 0 1 0 a 0 −a 1 0 0 0 C B −1 = B B a C 1 @ 0 0 0 a A 1 1 −a 0 a 0 9 I really do like Your Christ But I do dislike Your Christians Why can’t Your Christians be more like Your Christ? - Mahatma Gandhi
  • 10. Dr. Ole Peter Smith Instituto de Matemática e Estatística Universidade Federal de Goiás ole@mat.ufg.br - http://www.ime.ufg.br/docentes/olepeter Data: 27/05/2009 Semestre: Curso: Engenharia de Computação Disciplina: Álgebra Linear Prova: II 1. 2 pts. Dado os vetores em relação ao base canônica, (e1 , e2 , e3 , e4 , e5 ), em R5 : v1 = (1, 1, 1, 1, 1)T v2 = (1, −1, 1, −1, 1)T v3 = (3, −1, 3, −1, 3)T v4 = (0, 1, 0, 1, 0)T (a) Mostre que: V = ger(v1 , v2 ) = ger(v3 , v4 ). Qual a dimensão do V ? (b) Escreve v1 e v2 como combinações lineares de v3 e v4 . (c) Escreve v3 e v4 como combinações lineares de v1 e v2 . Solution: 2 pts. Dado os vetores em relação ao base canônica, (e1 , e2 , e3 , e4 , e5 ), em R5 : v1 = (1, 1, 1, 1, 1)T v2 = (1, −1, 1, −1, 1)T v3 = (3, −1, 3, −1, 3)T v4 = (0, 1, 0, 1, 0)T (a) Mostre que: V = ger(v1 , v2 ) = ger(v3 , v4 ). Qual a dimensão do V ? (b) Escreve v1 e v2 como combinações lineares de v3 e v4 . (c) Escreve v3 e v4 como combinações lineares de v1 e v2 . Solução: (a) Formamos o matriz com os vetores em colunas: 0 1 0 1 0 1 1 1 3 0 1 1 3 0 1 1 3 0 B B 1 −1 −1 1 C B C B 0 −2 −4 1 C B C B 0 −2 −4 1 C C V=B 1 1 3 0 C∼B 0 0 0 0 C∼B 0 0 0 0 C B C B C B C @ 1 −1 −1 1 A @ 0 −2 −4 1 A @ 0 0 0 0 A 1 1 3 0 0 0 0 0 0 0 0 0 Dai segue: dim ger(v1 , v2 , v3 , v4 ) = dim ger(v1 , v2 ) = dim ger(v3 , v4 ) = 2, o que implica: ger(v1 , v2 , v3 , v4 ) = ger(v1 , v2 ) = ger(v3 , v4 ) = 2. A dimensão é o posto do matriz V, isto é 2. (b) Omitindo as linhas com somente zeros: „ ˛ « „ ˛ « „ ˛ 1 « 1 1 ˛ 3 0 1 1 ˛ 3 0 1 0 ˛ 1 ˛ ∼ ˛ ∼ ˛ 2 0 −2 ˛ −4 1 0 1 ˛ 2 −1 2 0 1 ˛ 2 −1 2 Segue: v3 = v1 + 2v2 , e: v4 = 1 v1 − 1 v2 . 2 2 (c) Trocando o ordem dos vetores: „ ˛ « „ ˛ 1 1 « „ ˛ 1 1 « 3 0 ˛ 1 1 1 0 ˛ 1 0 ˛ ˛ ∼ ˛ 3 3 ∼ ˛ 3 4 3 −4 1 ˛ 0 −2 −4 1 ˛ 0 −2 0 1 ˛ 3 −2 3 1 4 1 2 Segue: v1 = v 3 3 + v , 3 4 e: v2 = v 3 3 − v . 3 4 2. 6 pts. Dado os vetores em relação ao base canônica, (e1 , e2 , e3 , e4 ), em R4 : v1 = (1, 1, 1, 1)T v2 = (1, −1, 1, −1)T v3 = (1, 1, −1, −1)T v4 = (1, −1, −1, 1)T (a) Mostre que os vi ’s são mutualmente ortogonais, isto é: vi · vj = 0 por i = j. (b) Encontrar um base ortonormal de R4 , (d1 , d2 , d3 , d4 ), tal que: di = ci vi . 10 Ditado.txt
  • 11. Dr. Ole Peter Smith Instituto de Matemática e Estatística Universidade Federal de Goiás ole@mat.ufg.br - http://www.ime.ufg.br/docentes/olepeter (c) Encontrar uma relação matricial entre os coordenados em relação aos ei ’s (antigos), xA , e os coordenados em relação aos di ’s (novos), xN : xA = D xN (d) Encontrar uma relação matricial entre os coordenados novos, xN , e os coordenados antigos, xA : xN = D xA (e) Justificar que vale: D = D−1 = DT = D. (f) Encontrar os coordenados dos vetores: w1 = (−1, 1, −1, 1)T w2 = (1, 2, 3, 4)T em relação ao base novo, (d1 , d2 , d3 , d4 ). Solution: 6 pts. Dado os vetores em relação ao base canônica, (e1 , e2 , e3 , e4 ), em R4 : v1 = (1, 1, 1, 1)T v2 = (1, −1, 1, −1)T v3 = (1, 1, −1, −1)T v4 = (1, −1, −1, 1)T (a) Mostre que os vi ’s são mutualmente ortogonais, isto é: vi · vj = 0 por i = j. (b) Encontrar um base ortonormal de R4 , (d1 , d2 , d3 , d4 ), tal que: di = ci vi . (c) Encontrar uma relação matricial entre os coordenados em relação aos ei ’s (antigos), xA , e os coordenados em relação aos di ’s (novos), xN : xA = D xN (d) Encontrar uma relação matricial entre os coordenados novos, xN , e os coordenados antigos, xA : xN = D xA (e) Justificar que vale: D = D−1 = DT = D. (f) Encontrar os coordenados dos vetores: w1 = (−1, 1, −1, 1)T w2 = (1, 2, 3, 4)T em relação ao base novo, (d1 , d2 , d3 , d4 ). Solução: (a) Verificamos: v1 · v2 = 1 − 1 + 1 − 1 = 0 v1 · v3 = 1 + 1 − 1 − 1 = 0 v1 · v4 = 1 − 1 − 1 + 1 = 0 v2 · v3 = 1 − 1 − 1 + 1 = 0 v2 · v4 = 1 + 1 − 1 − 1 = 0 v3 · v4 = 1 − 1 + 1 − 1 = 0 1 QED . √ √ (b) Normalizamos os vi ’s: |v1 | = |v2 | = |v3 | = |v4 | = 12 + 12 + 12 + 12 = 4 = 2. Assim: „ «T v1 1 1 1 1 d1 = = , , , |v1 | 2 2 2 2 „ «T v 1 1 1 1 d2 = 2 = ,− , ,− |v2 | 2 2 2 2 1 Quod Erat Demonstrandum 11 Ditado.txt
  • 12. Dr. Ole Peter Smith Instituto de Matemática e Estatística Universidade Federal de Goiás ole@mat.ufg.br - http://www.ime.ufg.br/docentes/olepeter „ «T v3 1 1 1 1 d3 = = , ,− ,− |v3 | 2 2 2 2 „ «T v4 1 1 1 1 d4 = = ,− ,− , |v4 | 2 2 2 2 É claro que di ’s são ortonormais, pois: di · dj = 0 por i = j, e: di · di = 1. (c) Organizamos os di ’s como colunas em uma matriz: 1 1 1 1 0 1 0 1 0 1 1 1 1 1 | | | | 2 1 2 2 2 B 2 −1 2 1 2 −1 2 C= 1B 1 C B −1 1 −1 C D = @ d1 d2 d3 d4 A = B 1 1 C @ −1 −1 A 2@ 1 1 −1 −1 A | | | | 2 1 2 2 2 2 −1 2 −1 2 1 2 1 −1 −1 1 Temos: 0 1 1 1 1 1 1B 1 −1 1 −1 C xA = @B Cx 2 1 1 −1 −1 A N 1 −1 −1 1 (d) Por D ser ortogonal, temos: D−1 = DT . Observamos que D ainda é simétrica: D = DT , ou seja: D−1 = D. Assim, invertemos a equação acima: 0 1 1 1 1 1 1B 1 −1 1 −1 C xN = @B Cx 2 1 1 −1 −1 A A 1 −1 −1 1 (e) Já justificamos isto no item anterior; por D ser ortogonal e simétrica, vale: D = D−1 = DT = D. (f) Encontramos w1 em coordenados novos: 0 10 1 0 1 0 1 1 1 1 1 −1 −1 + 1 − 1 + 1 0 1B 1 B −1 1 −1 C B 1 CB C = 1 B −1 − 1 − 1 − 1 C = B −2 C C B C B C 2@ 1 1 −1 −1 A @ −1 A 2 @ −1 + 1 + 1 − 1 A @ 0 A 1 −1 −1 1 1 −1 − 1 + 1 − 1 0 Ilustrando que: w1 = −2d2 . Para w2 : 0 10 1 0 1 0 1 1 1 1 1 1 1+2+3+4 5 1 B 1 −1 1 −1 C B C= 1 2 C B 1 − 2 + 3 − 4 C B −1 C @ 1 + 2 − 3 − 4 A = @ −2 B CB B C B C 2 @ 1 1 −1 −1 A @ 3 A 2 A 1 −1 −1 1 4 1−2−3+4 0 Podemos verificar: w2 = 5d1 − d2 − 2d3 . 3. 2 pts. (Cabeludo?) Ortogonalização de Graham-Schmidt Dado os vetores em R3 : v1 = (1, 1, 1)T v2 = (1, −1, 1)T v3 = (1, 1, −1)T (a) Mostre que v1 , v2 , v3 não são mutualmente ortogonais. (b) Mostre que v1 , v2 , v3 são linearmente independentes. (c) Escolhendo: d1 = v1 e d2 = v2 + αd1 , mostre que por: d1 · v2 α=− d1 · d1 obtemos um vetor, d2 ⊥ d1 . Encontrar d2 . 12 Ditado.txt
  • 13. Dr. Ole Peter Smith Instituto de Matemática e Estatística Universidade Federal de Goiás ole@mat.ufg.br - http://www.ime.ufg.br/docentes/olepeter (d) Escolhendo: d3 = v3 + βd1 + γd2 , mostre que por: d1 · v 3 d2 · v 3 β=− γ=− d1 · d1 d2 · d2 obtemos um vetor, d3 ⊥ d1 e d3 ⊥ d2 . Encontrar d3 . Solution: 2 pts. (Cabeludo?) Ortogonalização de Graham-Schmidt Dado os vetores em R3 : v1 = (1, 1, 1)T v2 = (1, −1, 1)T v3 = (1, 1, −1)T (a) Mostre que v1 , v2 , v3 não são mutualmente ortogonais. (b) Mostre que v1 , v2 , v3 são linearmente independentes. (c) Escolhendo: d1 = v1 e d2 = v2 + αd1 , mostre que por: d1 · v2 α=− d1 · d1 obtemos um vetor, d2 ⊥ d1 . Encontrar d2 . (d) Escolhendo: d3 = v3 + βd1 + γd2 , mostre que por: d1 · v 3 d2 · v 3 β=− γ=− d1 · d1 d2 · d2 obtemos um vetor, d3 ⊥ d1 e d3 ⊥ d2 . Encontrar d3 . Solução: (a) Os vi ’s não são mutualmente ortogonais, pois: v1 · v2 = 1 − 1 + 1 = 1 = 0 v1 · v3 = 1 + 1 − 1 = 1 = 0 v2 · v3 = 1 − 1 − 1 = −1 = 0 (b) 0 1 0 1 0 1 | | | 1 1 1 1 1 1 V = @ v1 v2 v3 A = @ 1 −1 1 A∼@ 0 −2 0 A | | | 1 1 −1 0 0 −2 Segue, que ρV = 3, ou seja: os vi ’s são linearmente independentes. (c) Na equação d2 = v2 + αd1 , fazemos o produto escalar com d1 : d1 · d2 = d1 · v2 + α(d1 · d1 ) Exigindo d1 ⊥ d2 - ou seja: d1 · d2 = 0 - segue: d1 · v2 α=− d1 · d1 Usando resultados anteriores: d1 · v2 = 1 d1 · d1 = 3 Assim: α = −1, 3 e assim: 13 Ditado.txt
  • 14. Dr. Ole Peter Smith Instituto de Matemática e Estatística Universidade Federal de Goiás ole@mat.ufg.br - http://www.ime.ufg.br/docentes/olepeter 0 1 0 1 0 2 1 1 1 3 1 d2 = @ −1 A − @ 1 A = @ − 4 A 3 3 2 1 1 3 Observamos: 0 1 0 2 1 1 3 2 4 2 d1 · d2 = @ 1 A · @ − 4 A = − + = 0 3 2 3 3 3 1 3 Isto é: d2 ⊥ d1 . (d) Na d3 = v3 + βd1 + γd2 fazemos o produto escalar com d1 e utilizamos que d1 · d2 = 0: d1 · d3 = d1 · v3 + β(d1 · d1 ) + 0 Exigindo d1 · d3 = 0, obtemos: d1 · v3 β=− d1 · d1 Fazendo o produto escalar com d2 , segue de mesmo maneira: d2 · v 3 γ=− d2 · d2 Usando resultados anteriores: d1 · v3 = 1 d1 · d1 = 3 Assim: 1 β=− 3 E mais: 4 d2 · v 3 = − 3 24 8 d2 · d2 = = 9 3 Assim: 1 γ= 2 Finalmente calculamos d3 : 0 1 0 1 0 2 1 0 1 1 1 1 1@ 1@ 3 A @ d3 = @ 1 A − 1 A+ −4 3 = 0 A 3 2 2 −1 1 3 −1 Observamos: 0 1 0 1 1 1 d1 · d3 = @ 1 A · @ 0 A=1+0−1=0 1 −1 0 1 0 2 1 1 3 d2 · d3 = @ 1 A · @ − 4 A= 2 +0− 2 =0 3 2 3 3 1 3 Isto é: d3 ⊥ d1 e d3 ⊥ d2 . 14 Ditado.txt